SlideShare a Scribd company logo
1 of 92
Ministry of Public Health Service of Ukraine
Ivano-Frankivsk National Medical University
Pathophysiology MODULE 1
GENERAL PATHOLOGY
Training-methodical manual
for class and out-of-class work for medical faculty students
Prepared by:
Gerasymchuk M. R.
Cherkasova V. V.
Zaiats L. M.
Ivano-Frankivsk, 2017
2
«The general nosology. Typical pathological processes»
Training-methodical manual for class and out-of-class work for
medical students / M.R. Gerasymchuk, V.V. Cherkasova, L.M. Zaiats //
IFNMU. Department of pathological physiology. – 2017. – 92 p. Discussed
and approved on the profile commission of medical&biological disciplines
meeting of Ivano-Frankivsk National Medical University.
Protocol № __ from «__» _______ 2017 year
3
CALENDAR PLAN OF PRACTICAL STUDIES
of pathological physiology for the students of the III course in the V semester
THEME OF PRACTICAL STUDIES DATES
1. The object, and tasks of pathophysiology. Methods of
pathophysiological investigations. Basic steps in pathological
physiology development.
04.09–08.09
2. General nosology. The doctrine of etiology and pathogenesis.
Pathogenic effect of ionizing radiation on the body.
11.09–15.09
3. The role of heredity and constitution in pathology. 18.09–22.09
4. Pathology of reactivity. Violation of immunological reactivity. 25.09–29.09
5. Hypersensitivity reactions of immediate and delayed type. 02.10–06.10
6. Cell pathophysiology. 09.10–13.10
7. Typical disorders of peripheral blood flow and microcirculation. 16.10–20.10
8. Inflammation: phlogogenic factors, pathogenesis of alteration,
mediators. Kongeim’s experiment.
23.10–27.10
9. Inflammation: pathogenesis of exudation and proliferation. 30.10–03.11
10. Disorders of thermal metabolism. Fever. 06.11–10.11
11. Tumors. 13.11–17.11
12. Starvation.Hypoxia. 20.11–24.11
13. Disorders of carbohydrate metabolism. Diabetes mellitus. 27.11–01.12
14. Disorders of lipid metabolism. Medical and social problems of obesity. 04.12–08.12
15. Disorders of water-electrolyte metabolism and microelements
metabolism.
11.12–15.12
16. Module 1 (practical part) 18.12–22.12
17. Module 2 (theoretical part) 25.12–29.12
The ESTIMATION FOR THE MODULE is defined as a sum of marks of current
educational activity (in points), which is proposed during the evaluation of theoretical
knowledges and practical skills. Maximal amount of points, which a student can collect -
200 points during of every module study, including for current educational activity – 120
points (together module topics are 120 points), on results final module control are 80 points.
Control of theoretical and
practical preparation
0 – 2 points – completely prepared homework;
0 – 5 points – oral answer;
0 – 1 points – test control during class.
Minimum – 0 points; positive – 4; maximum – 8 points
4
Topic № 1. The object, and tasks of pathophysiology. Methods of
pathophysiological investigations. Basic steps in pathological physiology
development. General nosology. Teaching about etiology and pathogenesis.
Pathogenic effect of radiation and electric current.
1. The The actuality of the theme. Pathological physiology is science, that
studies the functional changes at a sick man and animals. It studies the most
general conformities to the law of origin, development, consequences of illness.
By experiment, we may reproduce and study on animals the separate models of
illnesses, violation of organs and systems for the cognition of basic conformities
to the law of development of illnesses of man. Consequently, the experiment is
the basic method of pathophysiology. 2. Duration of the class – 1 hour 30 min.
3. Aim:
To know such terms as the «modelling», «experiment».
To be able:
a) The modeling of various forms of pathologic processes, protective and
adaptive reactions of humans;
b) Experimental therapy as an important method of studying and introducing
the new ways of treatment;
c) Clinical studying of various diseases with functional, biochemical,
immunological and other tests due to pathophysiology groundation therapy.
To perform practical work:
1. Main features and purpose of experiment in pathophysiology. Correlation
of method of clinical supervision with the pathophysiological experiment.
2. Possible experiment on a human? Mental and ethical aspects of
pathophysiological experiment.
3. Causes and consequences constantly change their places. The cause
(etiological factors) causes the pathologic reactions (process) and then these
reactions return to the first agent (etiological factor) and intensify it.
4. To analyse the “vicious circle” in the pathogenesis. Causes and
consequences constantly change their places.
4. Basic level.
The name of the previous and future disciplines The receiving of the skills
1. histology
2. biochemistry
3. physiology
4. internal medicine
5. reanimatology
6. radiology
Ionizing radiation. Main
characteristics. Physical and chemical
ionizing radiation interchange in
organism. Gas composition of air and
partial pressure of oxygen. Functional
units structure of respiratory system,
blood, blood circulation system.
5. Control questions of the theme:
1. Term definition – what is pathophysiology? Explain role in medicine.
5
2. History of pathophysiology development. Significance of scientific
works of K. Bernar, R. Virchow, U. Kongeim, I. I. Mechnikov, G. Selie and
other famous investigators. The origin of pathological physiology as a scientific
discipline. Formation and development of pathological physiology in Ukraine.
Scientific schools of pathophysiologists, the main directions of their activity.
3. The connect between the pathophysiology and other disciplines. General
pathophysiology, special or systemic and clinical pathophysiology.
4. Methods of pathological physiology. An experiment as the main method
of pathophysiology, its significance for solving fundamental problems of
medicine.
5. Name the methods and the aims of pathophysiology. Gist of experimental
method. Modeling pathological processes and diseases on animals: possibilities
and limitations. Significance of comparative-evolutional method for
development of pathophysiology.
6. Rules of work with laboratory animals. Types of experiment. General
principles of planning the experimental investigations, count, statistic processing
and analysis of the results.
7. Definition a notion “experiment”, peculiarities, significance and lacks of
acute experiment, peculiarities of chronic experiment. Experimental therapy.
8. General doctrine of disease. The basic concepts of general nosology.
Norm, health, disease (definition of WHO). A pathological process. A
pathological condition, pathological reaction.
9. Disease as a biological, medical and social problem. The abstract and the
concrete in the concept “disease”. Unity of the destructive and protective in
disease. Principles of disease classification, classification of WHO. The main
laws of a disease course.
10.Stages of disease development. Remission, relapse, complication.
Variants of disease outcome: complete and incomplete recovery.
11.Terminal conditions: pre-agony, agony, clinical death.
Pathophysiological bases of reanimation.
12. The main directions in the development of doctrine of disease: humoral
(Hippocrates), cellular (R. Virchow). Their development in the modern stage.
13. Definition of the concept “etiology”. Problem of causality in pathology.
The role of causes and conditions in occurrence of diseases. The main directions
in the development of doctrine of etiology: monocausalism, conditionalism,
constitutionalism, psychosomatic conception, etc.
14.Modern concepts of causality in pathology. Classification of etiological
factors. External and internal etiological factors. The concept of risk factors.
“Diseases of civilization”. Ecological, genetic, accumulative and ontogenetic
conceptions of occurrence of human diseases. Etiotropic principles of prevention
and treatment.
6
15.Definition of the concept “pathogenesis”. The destructive and adaptive
phenomena of pathogenesis. Manifestations of injury at different levels: a
molecular, cellular, tissue, organ, organism one.
16. Protective reactions of adaptation. Adaptation, compensation.
Mechanisms of immediate and long-term adaptation. The role of nervous and
humoral factors in their realization. “Compensatory-adaptative reactions”, types
of compensatory-adaptative reactions, role of compensatory-adaptative reactions
in convalescence mechanisms.
17. The cause-effect relations in pathogenesis. Variants of direct cause-
effect relations. “Circulars vicious”. The main link of pathogenesis. Role of the
local and general in pathogenesis.
18. Concept of localization and generalization of pathological processes.
Ways of their distribution. Specific and nonspecific mechanisms of pathogenesis.
Pathogenetic principles of disease treatment.
19.Pathogenic effect of ionizing radiation. Types of ionizing radiation.
Radiosensitivity of tissues. Mechanisms of direct and indirect radiation damage
of biological structures. Water radiolysis. Radiotoxins.
20.Manifestations of radiation damages on molecular, cellular, tissue, organ
and system levels.
21.Pathogenesis of radiation sickness, its main forms and syndromes. Early
and late effects of large and small doses of ionizing radiation.
22.Natural mechanisms of antiradiation protection. Pathophysiological bases
of radioprotection.
23.Influence of low atmosphere pressure upon organism, man activity in
condition of low atmosphere pressure, mechanisms of low atmosphere pressure
action, manifestations.
24.Influence of high atmosphere pressure upon organism, man activity in
condition of high atmosphere pressure, mechanism of high atmosphere pressure
action, manifestations of saturation and desaturation, hyperbaric oxygenation
use in medicine.
25.Pathogenic influence of electric current.
6. Independent audience work of student.
Protocol № 1 Date_____________________
Experimental work 1. Determining the amount of haemoglobin. Solution
of hydrochloric acid pours in a test-tube of hemometer to the number 2 on the
scale. Then collects 0.02 ml blood in capillary and outpour it in a test tube.
Mixture leaves for 5 minutes. After distilled water pour full in the test-tube until
the color of liquid in a test tube will be evened with the color of standard
solution. Calculate the amount of haemoglobin in mmol/l. Formula of
7
calculation: B • 0,6206, where B – is an amount of haemoglobin in g%; 0,6206 –
is a coefficient of count in unit of SI.
Conclusion:____________________________________________________________
Experimental work 2. Determining the amount of erythrocytes. In a test-
tube pour 4 ml of a 3% solution of chloride of sodium. By a capillary pipette
collect 0,02 ml blood and produce it on the bottom of test tube. The contents is
carefully mixed. Then drop of liquids by pipette place under preliminary
grinding (rubbing) in integumentary (covered) small glass of account chamber.
Count up erythrocytes in 5 large (that in 80 small) squares of net of Goryaeva
and calculate their amount in 1 litr of blood after a formula:
lТ
ААА
/
100
10
100
10
80
2004000 128


where: A – is an amount of red corpuscles
in 5 large squares; 4000 – the volume of small square makes 1/4000 mm3; 200 –
is dilution of blood; 80 – is an amount of the counted up small squares; 108 is a
multiplier for the count of amount of red corpuscles in unit of SI; T – 1012.
Conclusion:____________________________________________________________
Experimental work 3. Count of amount of leucocytes. In a test-tube pour
0,4 ml of a 3% solution of vinegar acid. By a capillary pipette collect 0,02 ml
blood and outpour it on the bottom of test-tube. Mixture is mixed, and then fills
the chamber of Goryaeva. Count up the amount of leucocytes in 100 large
squares of net. Expect the amount of leucocytes in 1 l of blood.
lG
АА
Х /
20
10
11600
204000 6



 , where: A – is an amount of leucocytes in 100
large squares; 1600 – is an amount of the calculated small squares;
4000
1
it is a
volume of small square in mm3; 20 – is a degree dilution of blood; 106 – is a
multiplier for the count of amount of leucocytes in unit of SI; G – giga - 109.
Conclusion:____________________________________________________________
8. Practice Examination.
Practice examination type 1. Choose the correct answer:
Test 1. Which scientist emphasized senescence of connective tissue cells
cytoplasm?
A. Bogomolets
B. Mechnikov
C. Dilman
D. Frolkis
E. Berdichev
Test 2. The people, living in endemic centre, has recovered the three-day
malaria. In one and a half year after moving in the other places occured
malaria once again. What the most probable form of this disease?
A. Superinfection
B. Reinfection
C. The Relapse
D. Persistent infection
E. Secondary infection
Test 3. Which one of the methods is the most important at
pathophysiology?
A. Epidemiological
B. Anatomical
C. Clinical
D. Experimental
Test 4. Through day after irradiation (the dose 3,5 Gr.) beside damaged
in shelters: erythrocytes-4,7 1012
/l, hemoglobin-9,0 mmol/l, leukocytes-11
109
/l, thrombocytes-270 109
/l. In daub shelters: basophils-1%, eosinophils-
3%, neutrophils-81%, lymphocytes-11%, monocytes-4%. For what period
sharp radiation sickness typical specified shifts?
A. The Primary reaction
B. The Peak disease
C. The Rumpled welfare
D. Reconstruction
E. Terminations
Test 5. Sudden death during dehermitizing of aircraft at height 19000 m
is rose owing to:
A. Carbonic acid deficit
B. Organism desaturation
C. Gas embolism
D. Oxygen deficit
Е. Height meteorism
Test 6. A 49-year-old man was suffering 12 years ago from rheumatic
myocarditis, endocarditis, and insufficiency of mitral valve. Examinations
showed the absence of inflammatory process, sufficient minute blood
volume. What is it?
A. Pathological reaction
B. Pathological process
C. Typical pathological process
D. Compensatory reaction
E. Pathological condition
Test 7. A 39-years-old patient has been suffering from gastric ulcer for
last 4 years. Pain in epigastric region, heartburn, nausea, and constipation
appear mainly in autumn and spring. Name this condition.
A. Remission
B. Acute period
C. Complication
D. Pathologic condition
E. Relapse
Test 8. Gasping respiration appears in a patient with severe lung
pathology. What terminal condition is this characteristic for?
A. Agony B. Pre-agony
9
C. Clinical death
D. Biological death
E. Terminal pause
Test 9. Respiratory standstill developed in a man as a result of action of
electric current from town mains for 0.1 seconds with position of electrodes
“right hand – head”. Indicate the most probable reason for this
complication.
A. Paralysis of inspiration center
B. Emotional stress
C. Paralysis of respiratory muscles
D. Reflex respiratory standstill (pain shock)
E. Total paralysis of respiratory center
Practice examination type 2 Give answer to the questions of the real-life
tasks:
Task 1. Experimentator should study mechanismas of dyspnea in case of
disorders of blood circulation in the lungs. Which main periods of
pathophysiological experiment he should make?
1 3
2 4
Task 2. In dog under anesthesia, was destroied cusp of mitral valve and in
this ways reproduce cardiac insufficiency. Which method of disease modeling
used?
______________________________________________________________________
Task 3. As a result of damage one of atomic electric station blocks arose
emissions of ionizing rays products. In zone of raised radiation activity found
oneself three men. Approximately they got on 250-300 Rad. They immediately
delivered in clinic.
1. What consequences do follow to wait in victim?
2. What periods pick out in development of radial sick?
3. What researches you should make for clearing up of lesion degree?
4. How would be differ state irradiated being received 50R? 500R? 6000R?
Answerfor the task: ____________________________________________________
__________________________________________________________________________
__________________________________________________________________________
__________________________________________________________________________
__________________________________________________________________________
__________________________________________________________________________
Practice examination type 3
I. Give the description of each form of radiation disease (1 – acute, 2 –
chronic):
10
Indicator 1/2 Indicator 1/2
A
The disturbance of hemo-
poiesis and blood system
(lymphopenia,
thrombocytopenia)
F Hemorrhagic syndrome
G Sexual dysfunction
B Anemia H Spasm paralytic syndrome
C Immune reactivity decrease J Shock
D
Dysfunction of the
alimentary tract, vomiting,
anorexia, diarrhea
K Asthenia
E Necrotic tonsillitis L Trophic disorders
II. For each statement choose T (true) or F (false) in the list provided.
№ Statement T F
1
The first period of acute radiation disease (ARD) with
duration from several hours to 1-2 days is characterized by
excitation, headache, instability of the vegetative functions,
lability of the arterial pressure and pulse
2
Latent period (one week) is accompanied by leukopenia
(progressing of lymphocytopenia, development of
granulocytopenia).
3
The third period is characterized by progressing leukopenia,
anemia. Hemorrhagic syndrome develops. Decrease of
immunologic reactivity.
4
The outcome of the ARD is multiple inflammatory processes
(necrotic angina, pneumonia, frontitis and others).
III. Give answer to the questions of the real-life task:
TASKS
1.
What direction of electricity through the human body is the most
dangerous?
A Head C Heart
B Kidney D Liver
2.
When the man is less sensitive to the electric current?
A Under narcosis C Hypoxia
B Tiredness D Alcohol intoxication
3.
The most resistant to electric current are all the below mentioned
except:
A External epidermal layer D Muscles
B Tendons and bones E Blood
C Nerves F Cerebrospinal fluid
Signature___________________
11
Literature:
Basic:
1. Robbins and Cotran Pathologic Basis of Disease 9thed./Kumar, Abbas, Fauto.–2015.–Ch.9.–P.426–432.
2. General and clinical pathophysiology. Edited by prof. A.V. Kubyskin. Simferopol. – 2011. – P. 17–85.
3. Pathophysiology, N.K. Symeonova. Kyiv, AUS medicine Publishing. – 2010. – P. 24–34.
4. Copstead Lee-Ellen C. Pathophysiology / Lee-Ellen C. Copstead, Jacquelyn L. Banasic // Elsevier Inc. –
2010. – P. 3–13, 1280–1283.
Additional:
1. Pathophysiology, Concepts of Altered Health States, Carol Mattson Porth, Glenn Matfin.– New York,
Milwaukee. – 2009. – P. 2–10.
2. J.B.Walter I.C.Talbot General pathology. Seventh edition. – 1996. – P. 1–17.
Topic 2: Role of heredity, constitution and aging changes in pathology.
1. The actuality of the theme. Genetic and congenital defects are important
at all levels of healthcare because they affect all age groups and can involve
almost any of the body tissues and organs. Congenital defects, sometimes called
birth defects, develop during prenatal life and usually are apparent at birth or
shortly thereafter.
About 2000 different hereditary diseases are known now. 4 % of new-born
suffer from these or other genetically conditioned defects. Heredity pathology
plays an important role in development of such hereditary conditioned diseases,
as atherosclerosis, essential hypertension, rheumatism, diabetes mellitus, and
gout.
2. Duration of the class – 1 h 30 min.
3. Aim:
To know the role of hereditary factors and constitution in the development
of diseases and pathological processes.
To be able: to analyze the pathogenesis of chromosomal diseases,
phenylketonuria.
To perform practical work: Characterize fragile X-associated mental
retardation syndrome produces a unique combination of phenotypic features that
affect the central nervous system, the testes, and the cranial skeleton.
4. Basic level.
The name of the previous and
future disciplines
The receiving of the skills
1. histology
2. biochemistry
3. physiology
4. pediatrics
Encoding of hereditary information.
Legitimacy and types of hereditary signs
transmission in generations.
Cytological genetic base.
5. Control questions of the theme:
1.Heredity as a cause and condition of disease development. Relation of the
hereditary and acquired in pathogenesis. Hereditary and congenital diseases.
Gene- and phenocopies. Classification of hereditary diseases.
12
2.Mutations. Principles of their classification. Causes of mutations.
Mutagenic factors of a physical, chemical and biological origin.
3.Mendelian disorders: diseases caused by single-gene defects:
• autosomal dominant (Huntington disease, neurofibromatosis type 1,
Marfan syndrome, Familial hypercholesterolemia (LDL receptor deficiency),
acute intermittent porphyria);
• autosomal recessive (Cystic fibrosis, sickle cell anemia, phenylketonuria,
Tay-Sachs disease (hexosaminidase A deficiency));
• X-linked (Fragile X syndrome, hypophosphatemic rickets, Duchenne
muscular dystrophy, Lesch-Nyhan syndrome, Glucose-6-phosphate
dehydrogenase deficiency, Hemophilia A and B, red-green color blindness,
Menke’s disease).
4.Monogenic hereditary diseases. Antigen associative diseases.
4a. Diseases caused by mutations in genes encoding structural proteins:
Marfan Syndrome, Ehlers-Danlos Syndromes
4b. Diseases Caused by Mutations in Genes Encoding Receptor Proteins or
Channels: Familial Hypercholesterolemia, Cystic Fibrosis;
4c. Diseases Caused by Mutations in Genes Encoding Enzyme Proteins:
phenylketonuria, Galactosemia; Lysosomal Storage Diseases: Tay-Sachs Disease
(GM2 Gangliosidosis: Deficiency in Hexosaminidase β Subunit), Niemann-Pick
Disease Types A, B and C, Gaucher Disease, Mucopolysaccharidoses; Glycogen
Storage Diseases (Glycogenoses): von Gierke disease, Pompe disease, McArdle
disease;
4d. Mitochondrial diseases: Leber hereditary optic neuropathy, MELAS:
mitochondrial encephalomyopathy, lactic acidosis, and stroke-like episodes),
myoclonic epilepsy with ragged red muscle fibers).
4e. Diseases caused by alterations of imprinted regions: Prader-Willi and
Angelman Syndromes.
5.Systems of anti mutational protection. Mechanisms of DNA reparation.
The role of disorders of reparative systems and “immune control” in an
occurrence of hereditary pathology.
6.Chromosomal diseases. Mechanisms of occurrence of the genome and
chromosomal mutations. Polyploidy, aneuploidy, deletion, duplication,
inversion, translocation.
6a. Cytogenetic disorders involving autosomes: trisomy 21 (Down
Syndrome), 22q11.2 deletion syndrome, trisomy 18 (Edwards syndrome),
trisomy 13 (Patau syndrome);
6b. Cytogenetic disorders involving sex chromosomes: Klinefelter
syndrome, Turner syndrome, “Super-woman” syndrome.
7.Congenital anomalies: polydactyly, cleft lip, cleft palate. Etiology.
Pathogenesis. Perinatal infections (TORCH Toxoplasma (T), rubella virus (R),
13
cytomegalovirus (C), herpesvirus (H), and any of a number of other (O)
microbes such as Treponema pallidum).
8.Methods of investigation, prophylaxis, and treatment of hereditary
diseases. Ways of correction of genetic defects. Outlooks of gene engineering.
9.Constitution, its role in pathology. Classification of constitutional types by
Hippocrates, Sigot, Kretchmer, I. P. Pavlov, A. A. Bogomolets. The concept of
diatheses.
6. Students’ practical activities.
Protocol № 2 Date_____________________
Experimental work 1. There is research of sexual chromatin (Bara's
bodies) in the epithelium cells of mucous layer of mouth. The impression
smear (touch smear) prepares from the mucous layer of mouth. The drop of a 1%
solution of acetoorcein on the smear. The drop is covered integumentary glass so
that paint evenly spread on the stroke and air under glass must be absent. A
surplus of paint is taken off by a filtration paper. Study preparation under the
small increase of microscope (lens×20, eyepiece×10). Find the accumulation of
cells and study it under immersion increase (lens×90, eyepiece×10). Interphase
nuclei are counted and mark how many they have contained the Bara's bodies.
Count up 100 cells. For an analysis, it is necessary to take away unharmed cells
with a round or oval nucleus. Sexual chromatin is placed under a nuclear
membrane, has a semilunar or triangle shape. For women – 28% all nuclei of
cells of epitheliums contain sexual chromatin, and for men - 0-1%.
Conclusion:____________________________________________________________
Experimental work 2. To define content of phenylpyruvic acid in urine.
For the exposure of phenylpyruvic acid in urine use the test-paper of type
“Biofan”, which are imbued by buffer solution of chlorous iron. The end of test-
paper is put into urine and through 30 sec. the results. A test is considered
positive if the test-paper becomes a green color.
Conclusion:____________________________________________________________
7. Practice Examination.
Practice examination type 1. Choose the correct answer:
Test 1. Which of the following characteristics are most typical of
multifactorial inheritance?
A. Sex predilection
B. Mitochondrial inheritance
C. Recurrence risks reflect the
number of affected relatives
14
D. Major cause of miscarriages E. Maternally derived
Test 2. In which variant of karyotype in the nucleus of somatic cells only
one Bar’s body can be found?
A. 47 XX, 15+
B. 45 XO
C. 45 XY
D.47 XY, 21+
E. 48 XXXY
Test 3. At examining of a patient-doctor stressed attention to her low
growth, wide shieldlike, insufficient physical and sexual development.
External genitalia formed on a feminine type, internal was underdeveloped.
Barr body in cells of mouth mucous membrane was not determined. For
what disease is it typical?
A. Klinefelter’s syndrome
B. Turner’s syndrome
C. Х-trisomy syndrome
D. Down’s illness
Е. Pathau’s syndrome
Test 4. Which of the following is most likely in an untreated child with
phenylketonuria (PKU)?
A. Elevated tyrosine
B. Increased skin pigmentation
C. Normal phenylalanine
hydroxylase levels
D. Elevated alanine
E. Decreased skin pigmentation
Test 5. The boy was born by woman 45 years with fission of the upper
lip ("cleft lip" and "cleft palate"). Significant breaches are discovered.
Under additional examination on the part of nervous, cardiovascular
systems and visions. At study, Karyotype is diagnosed trisomy on 13
chromosomes. What syndrome exists beside boy?
A. Klinefelter
B. Edwards
C. Turner
D. Patau
E. Down
Practice examination type 2. Give answers to the questions of the
picture and real-life task:
A young woman is referred for genetic counseling. She has a 3-year-old boy
with developmental delay and small joint hyperextensibility. The pediatrician has
diagnosed fragile X-associated mental retardation. She is currently pregnant with
her second child at 14 weeks of gestation. The family history is unremarkable.
1. What is the genetic mutation responsible for fragile X-associated mental
retardation? How does it cause the clinical syndrome of developmental delay,
joint hyperextensibility, large testes, and facial abnormalities?
2. Which parent is the probable carrier of the genetic mutation? Explain
why this parent and the grandparents are phenotypically unaffected.
3. What is the likelihood that the unborn child will be affected?
Answer for the task: ____________________________________________
______________________________________________________________________
______________________________________________________________________
15
______________________________________________________________________
______________________________________________________________________
______________________________________________________________________
______________________________________________________________________
______________________________________________________________________
Practice examination type 3
Picture. Inpute in the empty spaces apropriate sypmtoms or fitures.
16
Practice examination type 4
For each statement write T (true) or F (false) in the blank provided
№ Statement T F
1 Hypertension is a multifactorial disease.
2 Fat intake is a risk factor for many diseases.
3
A multifactorial trait is expressed when multiple genes and
environmental influences blend together.
4
In Down’s syndrome, the pathology is manifested
independently of an environment.
5
It is easy to distinguish between the effects of shared
environmental factors and the effects of common pool genes.
6 Psychiatric disorders do not manifest familial patterns.
7 Relative risk is a ratio between incidence and individuals.
8
Early type II diabetes may develop when an individual’s diet
changes to heavy carbohydrate consumption.
9
Finding and understanding environmental factors that affect
penetrance of specific genes is important if chronic familial
diseases are to be prevented.
10
A variation in the phenotype for different genotypes caused by
the environmental factors is a threshold liability trait.
11
The frequency of genetic disease in the population depends on
phenotypes.
12
Risk factors, when removed or eliminated, delay or prevent
disease.
13 A proband is the individual beginning the pedigree.
14
The recurrence risk is less when more than one sibling is
affected.
15
The existence of a particular risk factor indicates an individual
will develop a specific disease.
16
The expression of a disease may require both an inherited
defect and environmental exposure.
17
Multifactorial diseases can change substantially from one
population to another because gene frequencies and
environments differ.
18 Dizygotic twins are identical.
19
Recurrence risk is higher if the disease is more severe in the
proband.
20
The prevalence rate is the number of individuals affected by a
disease.
21
The incidence rate is the number of persons who have died
from the disease.
17
Signature___________________
Literature:
Basic:
1. Robbins and Cotran Pathologic Basis of Disease 9th international ed./ V.Kumar, A.K.Abbas, J.C.Aster –
2015. – Chapters 5, 10. – P. 137-182, 451–480.
2. Robbins and Cotran Pathologic Basis of Disease 9thed./ Kumar, Abbas, Fauto. – 2013.–Ch6.–P. 215–268.
3. General and clinical pathophysiology. Ed. by prof. A.V. Kubyskin. Simferopol. – 2011. – P. 86–104.
4. Copstead Lee-Ellen C. Pathophysiology / Lee-Ellen C.Copstead, J.L.Banasic//Else.Inc.–2010.–P.86–127.
5. Essentials of Pathophysiology: Concepts of Altered Health States (Lippincott Williams & Wilkins),
Trade paperback (2003) / Carol Mattson Porth, Kathryn J. Gaspard. Chapters 3-4– P. 36 – 62.
6. Symeonova N.K. Pathophysiology. – N.K. Symeonova. Kyiv, AUS M-ne Publishing. – 2010. – P. 44–65.
7. Pathophysiology, Concepts of Altered Health States, C.M.Porth, G. Matfin.– NY,M. –2009.–P.133–156.
Additional:
8. Gozhenko A.I. General and clinical pathophysiology / A.I. Gozhenko, I.P. Gurcalova // Study guide for
medical students and practitioners. Edited by prof.Zaporozan, OSMU. – Odessa. – 2005.– P. 49–60.
9. Silbernagl S. Color Atlas of Pathophysiology / S. Silbernagl, F. Lang // Thieme.St. NY. – 2000. – P. 2–9.
10. J.B.Walter I.C.Talbot General pathology. Seventh edition. – 1996. – P. 59–87.
Topic 3. Pathology of reactivity. Disorders of immune reactivity. AIDS.
1. The actuality of the theme. Reactivity is the characteristic of the
organism to react in a certain way on the influence of the environment. It is the
same as growing up, feeding, and metabolism.
Any pathological process in one or another degree changes the reactivity of
the organism and in the time the changing reactivity which exceeds physiological
of the organism can become a main development of the disease.
2. Duration of the class – 1h 30min.
3. Aim: to know types of reactivity.
To be able: to analyze the specific and non-specific reactivity.
To perform practical work: Resistance. Types. Interaction with reactivity.
4. Basic level.
The name of the previous and
future disciplines
The receiving of the skills
1. histology
2. biochemistry
3. physiology
4. immunology
5. internal medicine
Structure of the blood-brain barrier and others
hystohaematic barriers.
Physiological reactivity indexes.
Structure of central nervous system, eye, thyroid
gland, internal ear, and testis.
5. The advice for students . Properties of Human Immunoglobulins
Class
Ig
Content
in blood
serum
mg/L
% of
total
Ig
level
Mole-
cular
mass
kD
Fixatio
n of
comple-
ment
Trans-
ference
hrough
the
placent
Function
IgG
gen.
8-16 80 150 ++ +
Secondary immune response,
protection from bacteria and
viruses displays antiviral,IgG1 65 150 ++ +
18
6. Control questions of the theme:
1.Definition of the concept of reactivity. Reactivity as a condition of disease
development. Types of reactivity.
2.Dependence of reactivity on sex, age, heredity, condition of immune,
nervous and endocrine systems. Influence of environment on the reactivity of
organism. Manifestations of reactivity on a molecular, cellular, tissue, organ,
system level and on the level of the whole organism.
3.Reactivity by Bogomolets. Explain term “physiological system of
connecting tissue” and name elements of this system by Bogomoletz. What is a
role of this system in the maintenance of organisms’ homeostasis?
4.The theory of Hans’a Seley of diseases of adaptation.
5.Reactivity and biological barriers.
6.Definition of the concept of resistance. Passive and active resistance. The
connection of resistance with reactivity.
IgG2 23 150 + + antitoxin, and antibacterial
properties; only Ig that
crosses the placenta;
responsible for protection of
newborn; activates
complement and binds to
macrophages
IgG3 8 150 +++ +
IgG4 4 150 - +
IgM 0.5-2 6 900 +++ - Primary immune response
Forms the natural antibodies
such as those for ABO blood
antigens; prominent in early
immune responses; activates
complement
IgA
plasm.
1.4-1.9 13 160 - - Predominant Ig in
bodysecretions,
such as saliva,
nasal and
respiratory
secretions, and
breast milk;
protects mucous
membranes
IgA
secr.
2-5 380 - -
Mucous
membrane
protection
IgD 0-0.4 0-1 180 - -
Membrane receptors found
on B lymphocytes; needed
for maturation of B cells
IgE
14-450
ng/ml
0.002 190 - -
Reagines, protection from
parasites.
Binds to mast cells and
basophils; involved in
parasitic infections, allergic
and hypersensitivity
reactions.
19
7.Mechanisms of nonspecific resistance. Biological barriers, their
classification, significance for a resistance of organism.
8.Humoral factors of nonspecific resistance of an organism to infectious
agents (lysozyme, C-reactive protein, interferons).
9.Complement system and its disorders.
10.Phagocytosis, its mechanisms. Disorders of phagocytosis: causes,
mechanisms, consequences. Chediak-Higashi syndrome.
11.Mechanisms of the normal immune response of a humoral and cellular
type, their disorders. Role of lymphocytes, macrophages, eosinophils,
neutrophils. Major Histocompatibility Complex Molecules: The peptide display
system of adaptive immunity.
12.Immunologic tolerance: central and peripheral.
13.Immunodeficiency, a definition of the concept, classification (WHO).
Bruton’s X-linked agammaglobulinemia, Common variable immunodeficiency,
Ig A deficiency, X-linked hyper-IgM Syndrome, DiGeorge’s syndrome, Severe
Combined Immunodeficiency, agammaglobulinemia (Swiss type), Wiscott-
Aldrich syndrome.
14.Pathogenesis of the main clinical manifestations of isolated and combined
disorders of functions of T- and B-lymphocytes. Causes, mechanisms of
development and types of initial immunodeficiencies. The role of physical,
chemical and biological factors in the development of secondary
immunodeficiencies (clinical examples).
15. Pathophysiological characteristics of the acquired immunodeficiency
syndrome (AIDS).
7. Students’ practical activities.
Protocol № 3 Date_____________________
Experimental work 1. To learn the violations of barriers function and
adaptations of an organism in the conditions of a convulsive attack. A test it
is carried out on two mise or young rats. On employment 1% solution of trypan
blue subcutaneously in animals, from a calculation 2 ml on 100 gr of mass. In 30
min 0,3 ml enter intraperitoneal camphorated oil to one of the animals. This
animal is placed under a glass hubcap and mark time of offensive of cramps,
their character and time of death. Animals are killed and studied organs, a
substance of brain and liquid eye.
Conclusion: ____________________________________________________________
______________________________________________________________________
______________________________________________________________________
______________________________________________________________________
20
Experimental work 2. An action of decreased atmospheric pressure
upon organism of a rat (experiment). Place animal under a glass bell. Acquaint
with a condition of the animal before experiment beginning (behavior, skin
color, breathing rate, a reaction to sound irritant). By the medium of pump
gradually diminish pressure under the bell. Observe change of animal condition.
Clock each supervision. After the beginning of first cramps in animal gradually
let air in and observes renewal of organism functions. Write down the
experiment results in the table and protocol of experiment according to the
scheme:
Animal
Beginning
of
experiment
Behavior
Breath
rate
Color of
visible
cutaneous
covering
Time of
offensive of
cramps
1.Adult
2.New
born
Conclusion: ____________________________________________________________
______________________________________________________________________
______________________________________________________________________
______________________________________________________________________
8. Practice Examination.
Practice examination type 1. Choose the correct answer:
Test 1. The deficient content of immunoglobulins was revealed in a
patient. What cells of immune system produce immunoglobulins?
A. Plasma cells
B. T-killers
C. B-lymphocytes
D. T-helpers
E. NK-killers
Test 2. The gene coding for HLA antigen is present on:
A. 6 q
B. 6 p
C. 16 p
D. 16 q
E. 21 p
Test 3. The complement component with opsonin activity is:
А. C1
В. С3а
C. C3b
D. С5а
Test 4. Various cells of the oral mucous membrane and antimicrobial
substances synthesized by these cells play an important part in the local
immunity of the oral cavity. Specify the key factors for the local immunity:
A. Eosinophils
B. B-lymphocytes
21
C. IgG
D. Macrophages
E. Secretory IgA
Test 5. Which cytokine promotes the proliferation of T and B
lymphocytes?
A. IFN-γ (interferon γ)
B. IL-2 (interleukin 2)
C. IL-4 (interleukin 4)
D. TNF-α (tumor necrosis factor α)
E.TGF-β (transforming growth
factor β)
Test 6. The immunoblot detected gp120 protein in the blood serum. This
protein is typical for the following disease:
A. Virus B hepatitis
B. HIV-infection
C. Tuberculosis
D. Syphilis
E. Poliomyelitis
Test 7. Blood analysis of a patient showed signs of HIV infection
(human immunodeficiency virus). Which cells does HIV-virus primarily?
A. Specialized nervous cells (neurons)
B. Mast cells
C. Cells that contain receptor IgM (B-lymphocytes)
D. Proliferating cells (stem hematoplastic cells)
E. Cells that contain receptor T4 (T-helpers)
Test 8. A pregnant woman was detected to have IgM to rubella virus.
An obstetriciangynecologist recommended therapeutic abortion due to the
high risk of teratogenic affection of the fetus. Detection of IgM was of great
importance as it is these specific immunoglobulins that:
A. Indicate recent infection
B. Penetrate placental barrier
C. Have the largest molecular weight
D. Are associated with anaphylactic reactions
E. Are the main factor of antiviral protection
Test 9. A 5-year-old child is diagnosed with Bruton syndrome (X-linked
agammaglobulinemia) that manifests itself in severe clinical course of
bacterial infections and absence of B lymphocytes and plasma cells. What
changes of immunoglobulin content can be observed in blood serum of the
child with immunodeficiency?
A. Increased IgA, IgM
B. Decreased IgA, IgM
C. Decreased IgD, IgE
D. Increased IgD, IgE
E. No changes
Test 10. A 13-year-old boy presents with eczematous rashes on his shins
and torso. Anamnesis states cases of otitis, pneumonia, and furuncles in the
patient. Blood test: platelets - 70 · 109
/l, low activity of T helper and T
22
suppressor cells, low IgM, with normal IgA and IgG. What immunodeficient
disease does this boy have?
A. DiGeorge syndrome
B. Louis-Bar syndrome (Ataxiatelangiectasia)
C. Severe combined immunodeficiency (Swiss type)
D. Wiskott-Aldrich syndrome
E. Chediak-Higashi syndrome
Practice examination type 2. Give answers to the questions of the real-
life tasks:
Task 1. An 18-year-old man presents with complaints of fever, facial pain,
and nasal congestion consistent with a diagnosis of acute sinusitis. His medical
history is notable for multiple sinus infections, two episodes of pneumonia, and
chronic diarrhea, all suggestive of primary immunodeficiency syndrome.
Workup establishes a diagnosis of common variable immunodeficiency.
1.What are the common infectious manifestations of common variable
immunodeficiency? 2. What are the underlying immunologic abnormalities
responsible for these infectious manifestations? 3. What other diseases is
this patient at increased risk for? 4. What treatment is indicated?
Answerfor the task 1: ___________________________________________________
__________________________________________________________________________
__________________________________________________________________________
__________________________________________________________________________
__________________________________________________________________________
__________________________________________________________________________
__________________________________________________________________________
Task 2. The patient was hospitalized with complaints on cough, rise of
temperature up to 38-39 C, weakness, and headache. Illness was related to
super cooling. Pneumonia was diagnosed after clinical research.
1. What was a direct cause of the disease? 2. What role in disease beginning
did cooling play? 3. What reactivity mechanisms did lead to organism resistance
reduce? 4. What are your recommendations for organism resistance rising to
catarrhal diseases?
Answer for the task 2: ___________________________________________
__________________________________________________________________________
__________________________________________________________________________
__________________________________________________________________________
__________________________________________________________________________
__________________________________________________________________________
__________________________________________________________________________
Signature___________________
Literature:
Basic:
1. Robbins and Cotran Pathologic Basis of Disease 9th international edition./ V.Kumar, A.K.Abbas,
23
J.C.Aster – 2015. – Chapter 6. – P. 186–200, 211–231, 237–263.
2. Robbins Basic Pathology 9th edition./ Kumar, Abbas, Fauto. – 2013. – Chapter 4. – P. 99–158.
3. General and clinical pathophysiology. Ed. by prof. A.V. Kubyskin. Simferopol. – 2011. – P. 209–233.
4. Pathophysiology, Concepts of Altered Health States, C.M.Porth, G.Matfin.– NY. – 2009. – P. 400–427.
5. Symeonova N.K. Pathophysiology. – N.K. Symeonova. Kyiv, AUS M-ne Publ. – 2010. – P. 65–84.
6. Copstead Lee-Ellen C. Pathophysiology / Lee-Ellen C. Copstead, Jacquelyn L. Banasic // Elsevier Inc. –
2010. – P. 203–221, 263–286.
7. Essentials of Pathophysiology: Concepts of Altered Health States (Lippincott Williams & Wilkins),
Trade paperback (2003) / Carol Mattson Porth, Kathryn J. Gaspard. Ch.7-8, 10. – P. 121 – 149, 178–189.
Additional:
1. Robbins and Cotran Pathologic Basis of Disease 8th edition./ Kumar, Abbas, Fauto. – 2007. – Chapter 5.
– P. 108–118, 152–165.
2. Silbernagl S. Color Atlas of Pathophysiology / S. Silbernagl, F. Lang // Thieme.. NY. – 2000. – P. 42–47.
3. Gozhenko A.I. General and clinical pathophysiology / A.I. Gozhenko, I.P. Gurcalova // Study guide for
medical students and practitioners. Ed. by prof. Zaporozan, OSMU. – Odessa. – 2005.– P. 42–48.
Topic 4. Immediate types of hypersensitivity reactions.
1. The actuality of the theme. The immediate-type allergy is often met
in practical activity of physicians of various specialties. That is because of the
great environmental pollution by industrial products, chemical matters, and
allergens of vegetable animal, bacterial, fungus origin and also due to a wide use
of various drugs. That type of allergy can develop suddenly. The severity of their
proceeding is various – from slight reactions to anaphylactic shock dangerous for
one’s life. Preventing and treatment of the allergy reactions is based on
knowledge about their development mechanisms.
2. Duration of the class – 1 h 30 min.
3. Aim:
To know that allergy is a complex of breaches, appearing in our organism by
the humoral immunological reactions. Both inflammation and allergy is a
protective response on the exo- and endogenous factor.
To be able: to analyze the pathogenesis of allergy by A.D. Ado.
To perform practical work: to analyze the mechanisms of
immunologically mediated disorders by Coombs and Gell (1968)
2. 4. Basic level.
The name of the previous and future
disciplines
The receiving of the skills
1. Histology
2. Biochemistry
3. Physiology
4. Immunology
5. Anaestesiology
Structure and function of organs and vessels.
Immunity and its mechanisms. Antigens and
antibodies, their structure and function.
Sensitizing.
5. The advice for students .
Classificaton of allergic reactions by Coombs and Gell (1968)
№ Type Prototype disorder Immune mechanism
1. Anaphylactic
type
Anaphylaxia, some forms of
bronchial asthma
Formation of IgE (cytotropic)
antibody → release of vasoactive
24
amines and other mediators from
basophils and mast cells.
2. Cytotoxic type Autoimmune hemolytic
anemia, erythroblastosis feta-
lis, cytotoxic reactions the
action of large doses ofBogo-
mglets’s ACS (antireticular
cytotoxic serum)
Formation of IgG, IgM → binds
to antigen on target cell surface
→ phagocytosis of target cell or
lysis by C8,9
3. Immune
complex disease
Arthus reaction serum
sickness, systemic lupus
erythematosus, certain forms
of acute glomerulonephritis
Ag + Ab →activated Co →
attracted complexes neutrophills
→ release of lysosomal enzymes
4. Cell – mediated
(delayer)
hypersensivity
Tuberculosis, contact
dermatitis, transplant
rejection
Sensitized thymus-derived
T-lymphocytes → release of
lymphokines and T-cell-mediated
cytotoxicity.
5. By Roight
Stimulating
allergic
reactions. Via
BAS, hormones,
mediators
Collagen diseases, connective
tissue diseases, rheumatoid
arthritis and other.
The cells, containing Ag, begin to
function intensively under the
influence of Ab. Than cells are
secreted hormones or mediators.
6. Control questions of the theme:
1. Allergy. Definition of the concept and general characteristics of allergy.
2. Allergy and immunity. Etiology of allergy, kinds of exo- and endogenous
allergens. The significance of hereditary and acquired factors in the development
of allergy.
3. Principles of classification of allergic reactions. General characteristics of
allergic reactions by Cumbs and Gell. Stages of pathogenesis of allergic
reactions.
4. Anaphylactic reactions: experimental models, main clinical forms.
Immune mechanisms of anaphylactic reactions, the role of mast cells in their
development. Active and passive anaphylaxis, pathogenesis of anaphylactic
shock.
5. Cytotoxic reactions: experimental modeling, main clinical forms.
Mechanisms of antibody-mediated diseases. Mechanisms of cytolysis. Examples
of diseases, cytotoxic: autoimmune hemolytic anemia, acute rheumatic fever,
Goodpasture syndrome, transfusion reaction, autoimmune thrombocytopenic
purpura; non-cytotoxic: Graves disease (hyperthyroidism), myasthenia gravis,
insulin-resistant diabetes, pernicious anemia.
6. Immune complex-mediated reactions, experimental modeling,
pathogenesis, clinical forms. Factors determining pathogenesis of immune
complexes. Immune complex damages, their local and general manifestations.
Examples of diseases: Systemic lupus erythematosus, rheumatoid arthritis,
25
poststreptococcal glomerulonephritis, serum sickness, Arthus reaction, Sjögren
syndrome.
7. Students’ practical activities.
Protocol № 4 Date_____________________
Experimental work. Anaphylactic shock in a guinea pig. Watching a
movie by the results of experiment and analysis of observed results.
Reproduction: 0.5 ml of horse serum was injected into a peritoneal cavity
of a guinea pig two weeks before the main experiment. In a day of experiment
guinea pig is fixed on a desk, then researcher cutes skin along medial line of the
neck, and separates jugular vein. After this, he injects 2.0 ml of the horse serum
into the jugular vein and students observe anaphylactic shock manifestation.
After animal death researcher separates cardiac-pulmonary complex. Pay
attention to the character of damage in the lungs. It is necessary to answer on
questions at prepare of protocol in part «Discussion».
1) What type of allergy is it? Explain. 2) What is the mechanism of animal
sensitization? 3) What is the role of biologically active substances in
anaphylactic shock manifestation? 4) What was the reason for animal death?
Explain why. 5) Is it possible to cause an anaphylactic shock by repeat injection
of horse serum if an experiment could be unsuccessful? Explain.
Through 1.5- 2 min. mark the first symptoms of shock: scratching of snout,
standing of wool, an anxiety of animal, cough, cyanosis of a snout. A next
symptom is departing of excrement and urine. Cramps, at first tonic (a guinea pig
falls on a side, quotation marks pronate, muscles are tense), and then clonic
(shallow cramps of extremities), liquid breathing, with large pauses.
Death comes in default of breathing and work of the heart is stored.
Determination of degree of shock:
- I stage (+) - characterized by standing of wool;
- II stage (++) is standing of wool, cough, scratching of snout;
- III stage (+++) --standing of wool, cough, scratching of snout, departing of
excrement and urine, cramp;
- IV stage (++++) -- standing of wool, cough, scratching of snout, departing
of excrement and urine, death.
It is convenient to analyze the dynamics of anaphylactic-type of allergy
using, as an illustration, the experimental parenteral injection of a heteroserum to
a healthy animal (twice with the two-week interval).
Conclusion: ____________________________________________________________
__________________________________________________________________________
__________________________________________________________________________
__________________________________________________________________________
26
8. Practice Examination.
Practice examination type 1. Choose the correct answer:
Test 1. Anaphylactic shock developed in a patient with botulism after
second injection of antitoxic antibotulinus serum mixture. What is the main
mechanism of anaphylaxis?
A. Interaction of T-lymphocytes with mediators
B. Interaction of antigen with IgM
C. Interaction of macrophages with antigens
D. Interaction of antigen with IgE
E. Interaction of T-lymphocytes with tissue basophils
Test 2. In the 1960s, it was quickly ascertained that Peace Corpus
workers sent to schistosome-endemic areas were exposed to massive initial
doses of cercariae before any protective immunity existed. In these
individuals, IgG antibodies developed in response to the developing worms,
and when the adults began their prodigious release of eggs into the
circulation, the patients suffered acute and potentially life-threatening
symptoms of fever, edema, arthralgia, and rush. Which of the following is
another condition that arises by a similar immunologic mechanism?
A. Atopic allergy
B. Arthus reaction
C. Goodpasture syndrome
D. Tuberculin reaction
E. Transfusion reaction
Test 3. Catarrhal inflammation of bulbar conjunctiva and nose mucous
membrane develop in patient every year in spring and early summer, when
trees and flowers are in blossom. Production of specific antibodies to pollen
underlies this syndrome. What cells activate and develop exocytosis in this
syndrome?
A. Neutrophils
B. Macrophages
C. Lymphocytes
D. Mast cells
E. Throbocytes
Test 4. It is known that bronchial asthma develops by mechanism of
immediate hypersensitivity, which includes 3 sequential stages:
A. Immunological, pathochemical, pathophisiological
B. Pathochemical, pathophisiological, immunological
C. Pathochemical, immunological, pathophisiological
D. Pathophisiological, immunological, pathochemical
E. Pathophisiological, pathochemical, immunological
Test 5. Man with the caries is subjected to constant sensitization by
streptococcus antigen. What disease can appear due to this etiological
factor?
A. Glomerulonephritis
B. Pancreatitis
C. Myocarditis
D. Pulpits
E. Periodontitis
Test 6. A child with diphtheria 10 days after injection of antitoxic
antidiphtherial serum has developed skin rash, accompanied by severe itch,
rising temperature up to 380C and joints pain. What is the cause of these
symptoms?
A. Delayed type of hypersensitivity
B. Anaphylactic reaction
C. Contact allergy
D. Atopia
E. Serum sickness
Practice examination type 2. Give answer to the questions of the real-life
task: The 36 year old worker was hospitalized to dermatological department of
clinic. He complains on rash skin hands and itch. The rash appeared two months
ago. An application skin test with nickel sulphate was positive. A test on of
macrophages migration inhibition with nickel is positive too.
1. What one does testify allergy nature of the disease?
2. Is there any base to recognize this disease as a delayed allergy?
Answerfor the task______________________________________________________
__________________________________________________________________________
__________________________________________________________________________
__________________________________________________________________________
__________________________________________________________________________
Practice examination type 3. Put number of these illustrations (1 to 6) in
the correct order as to how anaphylaxis develops with brief explanation of
each step.
28
Signature___________________
Literature:
Basic:
1. Robbins and Cotran Pathologic Basis of Disease 9thed./Kumar, Abbas, Fauto.–2013.– Ch.4.–P.109–117.
2. General and clinical pathophysiology. Edited by prof. A.V. Kubyskin. Simfer. – 2011. – P. 233–257.
3. Copstead Lee-Ellen C. Pathophysiology / Lee-Ellen C. Copstead, Jacquelyn L. Banasic // Elsevier Inc. –
2010. – P. 222–241.
4. Pathophysiology, Concepts of Altered Health States, Carol Mattson Porth, Glenn Matfin.– New York,
Milwaukee. – 2009. – P. 410–424.
5. Symeonova N.K. Pathophysiology. – N.K. Symeonova. Kyiv, AUS medicine Publ. – 2010.–P.87–100.
Additional:
1. Robbins and Cotran Pathologic Basis of Disease 8thed./Kumar, Abbas, Fauto. –2007.–Ch.7.–P.119–152.
2. Gozhenko A.I. General and clinical pathophysiology / A.I. Gozhenko, I.P. Gurcalova // Study guide for
medical students and practitioners. Edited by prof. Zaporozan, OSMU. – Odessa. – 2005.– P. 96–104.
3. Essentials of Pathophysiology: Concepts of Altered Health States (Lippincott Williams & Wilkins),
Trade paperback (2003) / Carol Mattson Porth, Kathryn J. Gaspard. Chapter 10. – P. 168 – 177.
4. Silbernagl S. Color Atlas of Pathophysiology / S. Silbernagl, F. Lang // Thieme. Stuttgart. New York. –
2000. – P. 52–59.
Topic 5: Reactions of hypersensitivity of a delayed type.
Autoimune diseases. Transplantology.
1. The actuality of the theme. The human immune network is a
multifaceted defense system that has evolved to protect against invading
microorganisms, prevent the proliferation of cancer cells, and mediate the
healing of damaged tissue. Under normal conditions, the immune response deters
or prevents disease. However, occasionally the inadequate, inappropriate, or
misdirected activation of the immune system can lead to debilitating or life-
threatening illnesses, typified by allergic or hypersensitivity reactions,
transplantation immunopathology, autoimmune disorders, and
immunodeficiency states. Preventing and treatment of the allergy reactions is
based on knowledge about their development mechanisms.
2. Duration of the class – 1 h 30 min.
3. Aim:
To know that allergy is a complex of breaches, appearing in our organism by
the cellular immunological reactions. Both inflammation and allergy is protective
response on the exo- and endogenous factor.
To be able:
1) determine the leading link of the cellular immune reactions pathogenesis
and cause-effect relations of separate pathogenetic mechanisms;
2) draw a conclusion about the character of pathological changes during
cellular immune reactions in organism and possible mechanisms of their
development on the ground of functional and biochemical investigation data.
To perform practical work: to analyse the mechanisms of cellular
immunologycally mediated disorders by Coombs and Gell (1968) and explain
stimulating allergic reactions by Roight.
29
By Roight – stimulating allergic reactions. Via BAS, hormones, mediators
the cells, containing Ag, begin to function intensively under the influence of Ab.
Than cells are secreted hormones or mediators. Ex. Collagen diseases,
connective tissue diseases, rheumatoid arthritis and other.
4. Basic level.
The name of the previous
and future disciplines
The receiving of the skills
1. histology
2. biochemistry
3. physiology
4. immunology
Structure and function of organs and vessels. Influence
of BAS on vessel wall. Cellular Immunity and its
mechanisms. Antigens and antibodies, their structure and
function.
5. Control questions of the theme:
1. Reactions of hypersensitivity of a delayed type: experimental modeling,
main clinical forms. Features of immune mechanisms. The role of lymphokines.
2. Autoallergic diseases. Causes and mechanisms of their development. The
role of autoallergic component in pathogenesis of diseases.
3. Pseudoallergic reactions.
4. The main principles of prophylaxis and treatment of allergic reactions.
Desensibilization.
5. The experimental modeling of pathology of the immune system.
6. Pathophysiological bases of transplantation of organs and tissues.
7. Immune tolerance, its types. Methods of experimental modeling of
immune tolerance.
8. Mechanisms of development of immune tolerance and their disorders.
9. Immune interrelations in system “mother-fetus”.
10.“Graft versus host” reaction, conditions of its development, acute and
chronic forms.
11.“Host versus graft” reaction, conditions of its development, acute and
chronic forms.
12.The main principles of immune stimulation and immune suppression.
13.V types of allergic reactions.
6. Students’ practical activities.
Protocol № 5 Date_____________________
Experiment 1. To learn displays and analyze the mechanism of
development of hypersensitiveness of delayed type into a rat.
For the receipt of hypersensitiveness of slow type animal, three days prior to
experience sensitize (introduction to the pillow of paw tuberculin Freund's
adjuvant in an amount 100 MCL. 3 times with an interval in two weeks. Before
24 hours to experiment enter intraperitoneally) anaphylaxis-provoking dose of
30
tuberculin (0.3 ml). On session for a rat under anesthesia open an abdominal
region, look after the displays of allergic reaction from the side peritoneum and
prepare strokes-imprints. Strokes dye after Romanovsky's stain. The dried
upstroke is fixed three minutes in the glass with a methyl alcohol, and then dyes
Romanovsky's stain during 20-25 min. The morphological displays of
hypersensitiveness of slow type study under the immersion increase of
microscope. In the strokes-imprints of experimental animals find plenty of
lymphocytes and monocytes. Comparison the strokes of animals of controls paint
out also.
The results of experience describe and sketch. To explain displays and
mechanism of development of hypersensitiveness of slow type into a rat on the
basis of findings facts.
Conclusion: ____________________________________________________________
__________________________________________________________________________
__________________________________________________________________________
7. Practice Examination.
Practice examination type 1. Choose the correct answer:
Test 1. Allergic diagnostic tests are used for the diagnosis of many
infectious diseases (tuberculosis, brucelosis, tularemia etc). Diagnosis is
confirmed if papula and redness appear in the place of the allergen
injection. Antigens interaction reaction is conditioned by:
A. IgE and lymphokines
B. IgM and macrophages
C. T-lymphocytes and
lymphokines
D. IgE and T-lymphocytes
E. IgM and tissue basophiles
Test 2. Thyrotoxicosis was diagnosed in a patient. Antithyroid
antibodies were found in his blood. Which type of allergic reaction is
observed at development of this disease?
A. Immune complex-mediated
B. Stimulating
C. Anaphylactic
D. Cytotoxic
E. Delayed type hypersensitivity
Test 3. Mycobacterium tuberculosis results in an intracellular bacterial
infection that provokes which one of the following immune responses?
A. Natural killer cytotoxic response
B. CD8-positive cytotoxic T cell response
C. Complement mediated lysis of infected cell
D. T helper 1 delayed type hypersensitivity response
E. Eosinophilia
Test 4. Hyperergic inflammation form of upper respiratory tract
(larynx, trachea, bronchi) develops at a 6-year-old child. Threat of the
respiratory impairment develops and then necessity of using anti-
31
inflammatory hormones occurs. Which hormone has anti-inflammatory
property?
A. Cortisol
B. Adrenaline
C. Growth hormone
D. Testosterone
E. Insulin
Test 5. An alloimmune disorder is:
A. Graft rejection
B. Insulin-dependent diabetes
C. Myxedema
D. All of the above are correct
E. None of the above is correct.
Practice examination type 2. Give answers to the questions of the real-
life tasks:
Task 1. Equal sizes skin allotransplant was engrafted to two groups of
rabbits. A herewith leucocytes suspension taken from rabbits-donors of
allotransplant was infected intravenously to one group rabbit 2 weeks before
transplantation. In what group of rabbits rejection of skin transplants will occur
first? Why?
Answerfor the task 1_____________________________________________________
______________________________________________________________________________________
__________________________________________________________________________
Task 2. Skin allotransplant 6 dm2 was engrafted to of rats. Rejection
occurred during first 14 days. 30 days after rejection of allotransplant the same
donor’s skin were engrafted to same animals. What will be life duration of
transplants after repeated engrafting? Explain, why?
Answerfor the task 3_____________________________________________________
__________________________________________________________________________
__________________________________________________________________________
Task 3. The 36 year old worker was hospitalized to dermatological
department of clinic. He complains on rash skin hands and itch. The rash
appeared two months ago. An application skin test with nickel sulphate was
positive. A test on of macrophages migration inhibition with nickel is positive
too. 1. What one does testify allergy nature of the disease?
2. Is there any base to recognize this disease as a delayed allergy?
Answerfor the task 4_____________________________________________________
__________________________________________________________________________
__________________________________________________________________________
Practice examination type 3: Rebus riddle. Solve the riddle to learn a
characteristic of a skin disorder.
32
__________________________________________________________________________
__________________________________________________________________________
Signature___________________
Literature:
Basic:
1. Robbins and Cotran Pathologic Basis of Disease 9thed./Kumar, Abbas, Fauto.–2013.–Ch.4.–P.122–139.
2. General and clinical pathophysiology. Edited by prof. A.V. Kubyskin. Simf. – 2011. – P. 233–249.
3. Symeonova N.K. Pathophysiology. – N.K. Symeonova. Kyiv, AUS medicine Publ. – 2010.–P.95-100.
4. Copstead Lee-Ellen C. Pathophysiology / Lee-Ellen C. Copstead, Jacquelyn L. Banasic // Elsevier Inc. –
2010. – P. 222–241.
5. Pathophysiology, Concepts of Altered Health States, Carol Mattson Porth, Glenn Matfin.– New York,
Milwaukee. – 2009. – P. 416–424.
Additional:
1. Robbins and Cotran Pathologic Basis of Disease8thed./Kumar, Abbas, Fauto.–2007–Ch.7.–P.119–131.
2. Gozhenko A.I. General and clinical pathophysiology / A.I. Gozhenko, I.P. Gurcalova // Study guide for
medical students and practitioners. Edited by prof. Zaporozan, OSMU. – Odessa. – 2005.– P. 96–104.
3. Essentials of Pathophysiology: Concepts of Altered Health States (Lippincott Williams & Wilkins),
Trade paperback (2003) / Carol Mattson Porth, Kathryn J. Gaspard. Chapters 10– P. 168 – 172.
4. Silbernagl S. Color Atlas of Pathophysiology / S. Silbernagl, F. Lang // Thieme. NY. – 2000. – P. 52–55.
Topic 6. Cell injury. General mechanisms of cell damage.
1. The actuality of the theme. Cellular injury can be caused by any factor
that disrupts cellular structures or deprives the cell of oxygen and nutrients
required for survival. The injury may be reversible (sublethal) or irreversible
(lethal) and is classified broadly as chemical, hypoxia (lack of sufficient
oxygen), free radical, or infectious. Cellular injuries from various causes have
different clinical and pathophysiologic manifestations.
33
Cellular death is confirmed by structural changes seen when cells are stained
and examined under a microscope. No biochemical indicators of cellular death
are universally applicable because we still do not know precisely what
biochemical functions must be compromised before a cell dies.2. Duration of
the class – 1h 30min.
3. Aim: to know the cell responses to injury and the major types of cellular
necrosis.
To be able: to analyze the main molecular mechanism of cell membrane
damage, cellular adaptations occurring in atrophy, hypertrophy, hyperplasia,
dysplasia, metaplasia and two pathways of apoptosis.
To perform practical work: describe the cell responses on injury and
mechanisms of apoptosis.
4. Basic level.
The name of the previous
and future disciplines
The receiving of the skills
1. histology
2. biochemistry
3. physiology
4. pathomorphology
5. surgery
Types and sources of radial energy. Properties of ionizing rays.
Use of X-rays and radioactive elements in popular equipment
and medicine. Main measures safety due to deal with X rays
and radioactive particles. Ecological catastrophes with
radioactive environment pollution. Blood cells and methods of
their count. A bioelectrical activity of central nervous system
and its registration. Fluid mosaic model of the plasma
membrane. Cell populations and cycle landmarks. Lipid
peroxidation. Erythrocytes in hypertonic, isotonic, and
hypotonic solutions.
1. The advice for students.
Structural-functional organization of a cell:
The nucleus contains genetic material of a cell.
The membrane provides the integrity of cell structures.
Lysosomes contain a wide spectrum of hydrolytic enzymes.
Mitochondria provide energy needs of a cell by synthesis of ATP.
Ribosomes carry out protein synthesis.
The endoplasmic reticulum is a membranous structure containing
ribosomes and detoxic enzymes.
Golgi complex accumulates and distributes proteins necessary for
construction of various structural elements of a cell.
Homeostasis is the concept of a dynamic steady state, turnover of bodily
substances that maintain physiologic parameters within narrow limits. Stressors
cause reactions that alter this dynamic steady state or homeostasis. Deviations
from normal values, or homeostasis, cause disease.
Necrosis is local cell death and involves the process of cellular self-digestion
known as autodigestion or autolysis. As necrosis progresses, most organelles are
disrupted and karyolysis, nuclear dissolution from the action of hydrolytic
34
enzymes, becomes evident. There are four major types of necrosis: coagulative,
liquefactive, caseous, and fat. Gangrenous necrosis is not a distinctive type of
cell death but refers to large areas of tissue death.
The two pathways of apoptosis differ in their induction and regulation, and
both culminate in the activation of "executioner" caspases. The induction of
apoptosis is dependent on a balance between pro- and anti-apoptotic signals and
intracellular proteins. The figure shows the pathways that induce apoptotic cell
death, and the anti-apoptotic proteins that inhibit mitochondrial leakiness and
cytochrome c-dependent caspase activation and thus function as regulators of
mitochondrial apoptosis.
6. Control questions of the theme:
1. Cellular injury. Characteristics of the concept of “injury”. Principles of
classification of cell injuries.
2. Pathology of signalization. Pathology of signal reception. Disorders of
secondary messеngers.
3. Damage to an executive apparatus of a cell. Pathochemical consequences
of damage to a cellular nucleus. Proteins of thermal shock, antioncogenes,
immediate genes of pre-early reaction.
4. Mechanisms and manifestations of damage to subcellular structures:
plasmatic membrane, mitochondria, endoplasmatic reticulum, lysosomes,
microtubules and microfilaments, nucleus and cytoplasm.
5. Cellular adaptation to stress. Hypertrophy, hyperplasia, atrophy,
metaplasia. Cellular and subcellular regeneration.
6. Molecular mechanisms of cell injury. The role of lipid mechanisms in the
pathogenesis of alteration: lipid peroxidation, activation of membrane
phospholipids and free fatty acids. Ischemia-reperfusion injury ischemia-
reperfusion injury.
7. Causes, mechanisms, and consequences of intracellular calcium
concentration increase.
8. Role of electrolyte-osmotic mechanisms in cell injury. Causes,
mechanisms, and consequences of disorders of electrolyte transport system in
cells.
9. Causes and development of intracellular acidosis in cell injury.
10.Mechanisms of protection and adaptation of cell. Protective and
compensative reactions for the renovation of intracellular homeostasis.
11. Antioxidant mechanisms of cells.
12. Mechanisms of apoptosis. The mitochondrial (intrinsic) pathway of
apoptosis, the death receptor (extrinsic) pathway of apoptosis, activation, and
function of caspases. Autophagy.
13. Necrosis: coagulative, liquefactive, gangrenous, gangrenous, caseous,
fat, fibrinoid.
14. Principles of prevention and pathogenetic therapy of cell injury.
35
7. Students’ practical activities
Protocol № 6 Date_____________________
Experiment work. To learn the damage of tissues basophile in rat under
effect with adrenalin. In the abdominal region of rat enter 5 ml of hemocoel. An
animal is killed by ether; good massage a stomach, dissect the abdominal region
and collect a liquid in the test-tube. To the got liquid add 0.1% alcoholic solution
of neutral red (from the calculation of 0.2 ml paint on 1 ml liquids). Carefully
mix and make strokes in which study the initial state of tissue basophile (lens
90, eyepiece10). A liquid which remained is poured out for 1 ml in 2 test
tubes: in the first (control) add 0.2 ml of physiological solution, in the second –
0.2 ml (20 μkg) adrenalin.
Conclusion:____________________________________________________________
__________________________________________________________________________
__________________________________________________________________________
__________________________________________________________________________
8. Practice Examination.
Practice examination type 1: Choose the correct answer:
Test 1. Activation of universal membrane structure injuring mechanism
occurs during reperfusion syndrome. This mechanism is referred to as:
A. Beta-oxidation of lipids
B. Oxidation of cytochromes
C. Microsomal oxidation
D. Knoop-Lienen cycle
E. Peroxidation of lipids
Test 2. Which ions accumulation in the cytoplasm of muscular cells
accounts for stable constriction of myofibrils?
A. Calcium
B. Sodium
C. Potassium
D. Magnesium
E. Hydrogen
Test 3. Which of the following cellular injury is reversible?
A. Hyaline deposits in Liver
B. Dystrophic calcification in
tissue
C. Coagulative necrosis
D. Cloudy swelling
E. All of the above
Test 4. In the process of metabolism in the human organism, the active
forms of oxygen are formed, including superoxide anion radical. With the
aid of which ferments this anion is inactivated?
A. Glucose reductase
B. Glucose peroxidase
C. Super oxide dimutase
D. Catalase
E. Peroxidase
Test 5. Apoptosis is inhibited by:
A. bcl-2
B. p53
С. ras
D. c-myc
E. p21
Practice examination type 2. Give answer to the questions of the real-life
task. The nurse in the Emergency Department is caring for a client who has
acute heart failure. The physician is writing orders for pharmacological
management, including diuretics. Which laboratory value is most important for
the nurse to check before administering medications to treat heart failure?
A. Platelet count.
B. Potassium.
C. Calcium.
D. White blood cell count.
Answerfor the task:_____________________________________________________
______________________________________________________________________
______________________________________________________________________
______________________________________________________________________
Signature___________________
Literature:
Basic:
1. Robbins and Cotran Pathologic Basis of Disease 9thed./ Kumar, Abbas, Fauto. – 2013. –Ch.1.–P.1–28.
2. General and clinical pathophysiology / Ed. by A.V. Kubyshkin – V: NK Publ. – 2011. – P. 134–165.
3. Copstead Lee-Ellen C. Pathophysiology / Lee-Ellen C. Copstead, Jacquelyn L. Banasik // Elsevier Inc,
4th ed. – 2010. – P. 30–84.
4. Pathophysiology, Concepts of Altered Health States, Carol Mattson Porth, Glenn Matfin.– NY,
Milwaukee. – 2009. – P. 99–109.
5. Corwin Elizabeth J. Handbook of Pathophysiology / Corwin Elizabeth J. – 3th edition. Copyright В. –
Lippincott Williams & Wilkins – 2008. – Chapter 1. – P. 3–35.
Additional:
1. Robbins and Cotran Pathologic Basis of Disease 8th ed./Kumar, Abbas, Fauto.–2007.–Chap.1–P. 1–30
2. Gozhenko A.I. General and clinical pathophysiology / A.I. Gozhenko, I.P. Gurcalova // Study guide for
medical students and practitioners. Edited by prof. Zaporozan, OSMU. – Odessa. – 2005.– P. 30–41.
3. Essentials of Pathophysiology: Concepts of Altered Health States (Lippincott Williams & Wilkins),
Trade paperback (2003) / Carol Mattson Porth, Kathryn J. Gaspard. – Сhapters 1-2. – P. 1–14, 28–35.
4. Silbernagl S. Color Atlas of Pathophysiology / S. Silbernagl, F. Lang // Thieme. NY. – 2000. – P. 2–13.
Topic 7: Pathophysiology of the micro- and macrocirculation.
1. The actuality of the theme. Blood vessels function in the delivery of
oxygen and nutrients to the tissues and in the removal of waste products from the
tissues. Unlike disorders of the respiratory system or central circulation that
cause hypoxia and impair oxygenation of tissues throughout the body, the effects
of blood vessel disease usually are limited to local tissues supplied by a
particular vessel or group of vessels.
Disturbances in blood flow can result from pathologic changes in the vessel
wall (i.e., atherosclerosis, vasculitides), acute vessel obstruction caused by
thrombus or embolus, vasospasm (i.e., Raynaud’s phenomenon), abnormal
vessel dilation (i.e., arterial aneurysms or varicose veins), or compression of
37
blood vessels by extravascular forces (i.e., tumors, edema, or firm surfaces such
as those associated with pressure ulcers).
2. Duration of the class – 1 hour 30 min.
3. Aim:
To know the basic factors of clot formation as the Virchow’s triad:
1. Injury to a vessel’s wall (by mechanic factors, electric current, chemical,
biological factors). These abnormalities also accompany atherosclerosis,
hypertension, and allergic process.
2. Disturbance of the balance between coagulation and fibrinolytic systems.
3. Slowing of blood flow and its abnormalities. It explains why thrombosis
of veins occurs 5 times more often than one of the arteries.
To be able: to study typical disorders of microcirculation, their significance,
and possible consequences.
To perform practical work: to analyze the pathogenesis of sludge
formation.
4. Basic level.
The name of the previous and
future disciplines
The receiving of the skills
1. histology
2. biochemistry
3. physiology
4. surgery
5. internal medicine
Structure of vascular stream. Vessels, which form the
microcirculation stream. Structure and functions of
endothelial cells, basal membrane. Two types of nitric
oxide (NO) synthesis in endothelium and macrophages.
The coagulation cascade. The- central roles of thrombin in
hemostasis and cellular activation. The central roles of
thrombin in hemostasis and cellular activation.
5. The advice for students.
Dynamics of Transcapillar Fluid Exchange (by Starling and Gayton)
Active powers
Arterial end of
capillary
mm Hg
Equilibrium point
mm Hg
Venous end of
capillary
mm Hg
Pressing out
Capillary hydrostatic
pressure
30.0 17.0 10.0
Attractive tissue pressure 5.3 5.3 5.3
Interstitial oncotic pressure 6.0 6.0 6.0
Total 41.3 28.3 21.3
Retentive
Oncotic pressure of plasma 28.0 28.0 28.0
Resulting In tissue 13.3 In tissue 0.3 In vessel 6.7
6. Control questions of the theme:
1.Definition of the concept of arterial hyperemia, etiology, and pathogenesis,
classification. External symptoms of arterial hyperemia.
2.Changes in tissues caused by arterial hyperemia, their significance, and
possible consequences.
38
3.Etiology and pathogenesis of venous hyperemia (congestion), etiology and
pathogenesis, external symptoms of venous hyperemia.
4.Changes in tissues caused by venous hyperemia, their significance, and
possible consequences.
5.Definition of the concept of ischemia. Etiology, pathogenesis of ischemia.
Changes in tissues at ischemia, their significance. Consequences and outcomes
of ischemia. External symptoms of ischemia.
6.The concept of reperfusion syndrome, ischemic toxicosis.
7.Definition of stasis. Etiology. Pathogenesis. Capillary (true) stasis.
8.Thrombosis as a reason for peripheral circulation disorder. Etiology and
pathogenesis.
9.Embolism as a reason for local circulation disorder. Stages and mechanisms
of embolism, types of emboli. The role of reflex mechanisms in the development
of common disorders caused by an embolism.
10. Embolism of systemic and pulmonary circulation; embolism of the portal
vein.
11. Typical disorders of microcirculation. Intravascular disorders of
microcirculation: changes of fluidity and rheological properties of blood.
12. Hemoconcentration, disturbances of suspension stability, aggregation and
agglutination of erythrocytes, sludge-phenomenon.
13. The role of aggregation of thrombocytes and disseminated intravascular
coagulation in development of microcirculation disorders (DIC-syndrome).
14. Disorders of tone, mechanical safety, and permeability of microvessels.
15. Extracellular disorders of microcirculation. Accumulation of
physiologically active substances, ions, edematous fluid in perivascular space.
16. Mesenchymal dystrophy. Capillarotrophic insufficiency.
17. Typical disorders of lymphodynamics: mechanical, dynamical and
resorption insufficiency of lymph circulation.
7. Students’ practical activities
Protocol № 7 Date_____________________
Experimental work 1. Venous hyperemia and venous stasis of the hand
(experiment). Measure up the arterial pressure. Fill the cuff with air so that
pressure in it becomes equal to diastolic pressure. Thus arterial blood float will
supply active, and venous outflow will decrease. Venous hyperemia will arise. 2
min after it is necessary to pay attention to visible symptoms and complaints due
caused by venous hyperemia. Then increase pressure in the cuff up to average
mean between systolic and diastolic. Venous stasis will arise. Arterial blood
inflow preserves but venous outflow stops. Ask the examined person about his
complaints, study skin color, skin temperature and sensitiveness, the condition of
39
superficial veins. Let the air out from the cuff. Note the results of examining in
the table and do protocol by a standard scheme.
Indexes Venous hyperemia Venous stasis
Complaints of the examined
Skin colour
Skin temperature
Skin sensitiveness
Condition of superficial veins
Conclusion: __________________________________________________________
Experimental work 2. Ischemic stasis and reactive arterial hyperemia of
the hand (experiment). To put a cuff, connected to sphygmomanometer on the
medial third of shoulder. Measure up the arterial pressure. To fill the cuff with
air so that pressure is 10 points higher than systolic pressure. Thus arterial and
venous blood flow stops. Ischemic stasis develops. Ask the examined person
about his complaints, study skin color, skin temperature and sensitiveness, the
condition of superficial veins. Let the air out from the cuff and observe arising
from reactive hyperemia. Note the results of examining in the table and do
protocol by a standard scheme.
Indexes Ischemic stasis Reactive hyperemia
Complaints of the inspected
Skin color
Skin temperature
Skin sensitiveness
Note the results of examining in the table and does protocol by standard
scheme explaining mechanism of every sign arise.
Conclusion: __________________________________________________________
Experimental work 3. Fatty embolism of the frog's tongues vessels
(experiment). Anesthetize the frog. Fix it on the board with the back down. Cut
its chest and open the heart. Put a cotton ball moistened with the solution of
Ringer on the heart. Take out the frog's tongue and straighten it upon the opening
in the board, watch the circulation of the vessels under the microscope. Carefully
inject 0,5-1,0 ml of fat emulsion into the ventricle of the heart. Continue to
40
observe the circulation in the vessels of the tongue. Draw the picture you
observed under the microscope. An answer to questions in a discussion:
1. What kind of typical disorders of regional blood circulation is it? 2. What
kind of embolism can be concerned the process which appeared during the
experiment? 3. Can fatty embolism of the brain appear when big tubular bones
are broken?
Conclusion: __________________________________________________________
8. Practice Examination
Test 1. Upper neck node of a sympathetic trunk was removed from the
rabbit on an experiment. Reddening and increased temperature of the skin
of the head are observed. What form of peripheral circulation of the blood
developed in the rabbit?
A. Venous hyperemia
B. Stasis
C. Neuroparalytic arterial
hyperemia
D. Metabolic arterial hyperemia
E. Neurotonic arterial hyperemia
Test 2. A 54-year-old man was admitted to the hospital with complaints
of pain in the right subcostal region, vomiting with blood. Objectively:
enlarged liver, varicose veins in the stomach and esophagus. Dysfunction of
what vessel is likely to be?
A. Vena hepatica
B. Vena cava superior
C. Vena cava inferior
D. Vena porta
E. Aorta abdominalis
Test 3. What are the mechanisms of aggregate formation?
A. Vascular wall damage.
B. Blood flow impairment.
C. Changes in protein composition of blood.
D. Quantitative and qualitative corpuscles.
E. All the above-mentioned factors.
Test 4. What does the term “sludge” mean?
A. Penetration of foreign bodies into the blood and their accumulation in it.
B. Formation of a large quantity of immunocomplexes in blood.
C. Accumulation and aggregation of blood cells in the microcirculatory bad.
D. An extreme degree of corpuscle aggregation.
E. Septicemia.
Test 5. After fast surgical removing of coronary artery occlusion in a
patient with ischemic heart disease, secondary injury of myocardium
develop (reperfusion syndrome) characterized by necrobiotic changes in the
focus of previous ischemia. This complication results from:
41
A. Excessive accumulation of calcium ions
B. Deficiency of potassium ions
C. Deficiency of adenosine triphosphate
D. Accumulation of hydrogen ions
E. Deficiency of creatine phosphate
Test 6. A patient has acute pain in his chest, dyspnea, tachycardia,
cyanosis, and decreased BP. Pulmonary infarction was diagnosed in this
patient. Which factor is the most common cause of pulmonary infarction?
A. Congestion in the pulmonary circulation
B. Embolism by thrombus from veins of lower extremities
C. Increase in number of platelets
D. Activation of fibrinolytic system
E. Pneumothorax
Practice examination type 2. Give answers for the real-life tasks (yes/no)
Task 1. Pick out microcirculatory vessels:
Indication y/n Indication y/n
A Large arteries. E Venules.
B Small arteries. F Arteriovenular
anastomoses (shunts).
C Arterioles. G Veins.
D Capillaries. H Aorta
Task 2. Which are the regulating mechanisms of microcirculatory vessels:
A Reflex. C Hemitic.
B Humoral. D Genetic.
Task 3. What is true (T), what is false (F) in the following:
Questions T F
A Intravascular blood clotting (IVBC) can be generalized
(disseminated) and local
B The process of a platelet plug formation could be divided into
two stages: the cellular stage and the plasma stage of
coagulation
C The stage of adherence, aggregation, and agglutination of
platelets and other blood cells is caused by the activating effect
of various aggregation stimulators (thrombin, factor of platelets
aggregation, serotonin, prostaglandins, thromboxane A2 and
others)
D The second stage is characterized with the formation of active
thrombin, which catalyzes transformation of fibrinogen into
fibrin with the formation of a clot
Practice examinationtype 3
42
Task 1. What is true (T), what is false (F) in the folowing functional
changes for arterial hyperemia
Questions T F
A Speeding up of blood flow in microcirculation bed
B Intensifying of metabolism and organ functioning
C Dilatation of small arteries, arterioles, capillaries
D Increasing the number of functioning vessels
E Hypoxia
Task 2. Match the correct answer what is physiological (a) and what is
pathologic (b) arterial hyperemia
Indication a/b Indication a/b
A Infectious rash F Heat hyperemia
B Ultraviolet erythema G Systemic lupus
erythematosus
C Hyperemia of the brain in
mental activity
H Hyperemia of the brain in
meningitis
D Hyperemia of pancreatic
gland and stomach in
digestion
J Hyperemia of skeletal
muscles in hard physical
work
E Typhoid fever K Inflammatory hyperemia
Task 1. Plaster bandage was imposed on the patient with the right humeral
bone fracture. Next day the soft-tissue swelling appeared, extremity became
cyanotic, the temperature of the injured hand skin decreased.
1. What violation of circulation of blood arose up? 2. Explain the
mechanisms of development of the soft-tissue swelling. 3. What is the
mechanism of skin cyanosis?
Answersfor the task 1. _________________________________________________
______________________________________________________________________
______________________________________________________________________
______________________________________________________________________
______________________________________________________________________
Task 2. Alpinists slowly rose on the slope of a mountain during 6 hours.
With every step, getting up was given all heavier. A general weakness,
palpitation, shortness of breath, syncope, head pain, a decrease in appetite,
meteorism ware marked.
1. What was the direct reason for these disorders at alpinists? 2. How is this
symptom-complex named? 3. Explain the mechanism of violations development.
4. At what height were alpinists on approximately? 5. What value has
tachycardia and tachypnoea in this situation, what are the mechanisms of their
development?
43
Answersfor the task 2. _________________________________________________
______________________________________________________________________
______________________________________________________________________
______________________________________________________________________
______________________________________________________________________
Task 3. Patient V. was in a surgical clinic with thrombophlebitis of right
lower extremity. After a careless movement the expressed shortness of breath,
pain in a thorax, cyanosis arose up.
1) Are these violations resulted by thrombophlebitis of lower extremity? In
what cases are such consequences of thrombophlebitis possible? What did arise
up at the patient? Is localization of complications at this patient casual? Explain.
2) Is development of complications of thrombophlebitis possible from the side
of other organs - brain, kidney, spleen?
Answersfor the task 3. _________________________________________________
______________________________________________________________________
______________________________________________________________________
______________________________________________________________________
______________________________________________________________________
Signature___________________
Literature:
Basic:
1. Robbins and Cotran Pathologic Basis of Disease 9th ed/ Kumar, Abbas, Fauto. –2013.–Ch 3.–P.75–97.
2. General and clinical pathophysiology. Edited by prof. A.V. Kubyskin. Simf. – 2011. – P. 105–120.
3. Pathophysiology, N.K. Symeonova. Kyiv, AUS medicine Publishing. – 2010. – P. 105–125.
4. Copstead Lee-Ellen C. Pathophysiology / Lee-Ellen C.Copstead, J.L.Banasic // Elsev.–2010.–P.347–371.
5. Pathophysiology, Concepts of Altered Health States, C.Mattson Porth,G.Matfin.–NY.–2009.–P.462–474.
Additional:
1. Robbins and Cotran Pathologic Basis of Disease 8th ed./Kumar, Abbas, Fauto. –2007.–Ch.4.–P. 81–100.
2. Gozhenko A.I. General and clinical pathophysiology / A.I. Gozhenko, I.P. Gurcalova // Study guide for
medical students and practitioners. Edited by prof. Zaporozan, OSMU. – Odessa. – 2005.– P. 60–67.
3. Essentials of Pathophysiology: Concepts of Altered Health States (Lippincott Williams & Wilkins),
Trade paperback (2003) / Carol Mattson Porth, Kathryn J. Gaspard. Chapters 15. – P. 263 – 272, 178–189.
Topic 8: Inflammation: phlogogenic factors, pathogenesis of alteration,
local and general sings. Kongeim’s experiment.
1. The actuality of the theme. Inflammation is the reaction of vascularized
tissue to local injury.
The causes of inflammation are many and varied. Inflammation commonly
results because of an immune response to infectious microorganisms. Other
causes of inflammation are trauma, surgery, caustic chemicals, extremes of heat
and cold and ischemic damage to body tissues. Inflammatory conditions are
named by adding the suffix – itis to the affected organ or system. For example,
appendicitis refers to inflammation of the appendix, pericarditis to inflammation
44
of the pericardium, and neuritis to inflammation of a nerve. More descriptive
expressions of the inflammatory process might indicate whether the process was
acute or chronic and what type of exudate was formed (e.g., acute fibrinous
pericarditis).
2. Duration of the class – 1h 30 min.
3. Aim:
To know: 1. Definition of the notion "inflammation". 2. Causes of the
inflammation. 3. The role of mediators in inflammation development. 4. The
meaning of the organism reactivity in inflammation development. 5. General
manifestation of inflammatory reaction. 6. Clinical symptoms of the
inflammation. 7. Causes and mechanisms of the vascular permeability disorder in
inflammation. 8. Methods of the vascular permeability study in the area of
inflammation.
To be able:
- to describe the processes of alteration, exudation and proliferation;
- to give description of mediators of inflammation and explain their role in
pathogenesis of inflammation;
- to reproduce inflammation in an experiment.
To perform practical work: to analyse of the pathogenesis of
inflammation: disorders and after effect.
1. Basic level.
The name of the previous
and future disciplines
The receiving of the skills
1. Histology
2. biochemistry
3. physiology
4. internal medicine
5. surgery
Functional parts of bloodstream. Conception about the
microcirculation. Mechanisms of regulation of blood
circulation in capillaries and venules. Conception about
the role of connecting tissue cells, their structure, main
functions. Conceptis about the products ofarachidonic
acid cascade, kalikrein-kinin system of blood,
complement system
5. The advice for students . Inflammation hallmarks
Systemic Local
Leukocytosis  redness (rubor)
 heat (calor)
 pain (dolor)
 swelling (tumor)
 loss of function
(functio laesa)
Leukopenia (in inflammation of viral origin)
Fever
Change of protein composition of blood: acute phase α-
protein (acute inflammation) and -globulins (chronic
inflammation)
Change of ferment composition of blood: increase of
transaminase, hyaluronidase, trombokinase activity
Increase of erythrocyte sedimentation
Change of hormone content:
 catecholamins
 corticosteroids
45
Immune system alterations and allergization of organism:
 antibody titre
 appearance of sensibilizing lymphocytes in blood
 development of allergic reactions
6. Control questions of the theme:
1.Definition of the concept “inflammation”. Etiology of inflammation.
Classification of the inflammatory agents.
2.Stages of inflammation. Cardinal symptoms of inflammation in
experiment (Cels, Galen). Classification of inflammation.
3.Acute inflammation. Primary and secondary alteration. Causes and
mechanisms of secondary alteration.
4.Mediators of inflammation. Their classification. Role of cytokines in
pathogenesis of inflammation.
5.Role of lysosomal enzymes, free radicals, peroxides and system of
complement in tissue injury.
6.Complement activation in the development of inflammatory process. Its
disorders.
7.Products of tissue basophile degranulation.
8.Derivates of arachidonic acid: prostaglandins, leukotrienes, thromboxanes.
9.Kallikrein-kinin system.
10.Biochemical and physicochemical disorders in focus of inflammation.
Local acidosis, hyperosmia, hyperoncia.
11.Methods of study of vascular permeability are in the hearth of
inflammation.
12.Cellular events: leukocyte recruitment and activation. Endothelial and
leukocyte adhesion molecules.
7. Students’ practical activities
Protocol № 8 Date_____________________
Experimental work. The change of the vessels permeability for
anaphylaxis-like inflammation (experiment). It is necessary to inject trypan
blue under the back skin of the rat in the quantity of 1-2 ml. 20 minutes after
causing the inflammation on the rear extremity using the method mentioned
above. After some time examine the skin color change of the animal and also
compare the color of injured and opposite extremities. Analyse the mechanism of
the permeability change. Write down the experiment results and protocol of
experiment according to a scheme. For a discussion of the results, it is necessary
to give an answer to questions.
1. Enumerate local signs of inflammation.
2. What do biological active substances appear in the area of inflammation?
46
3. How and why the permeability of the vessels in the area of the
inflammation does change?
4. What changes in the metabolism to promote the development of the
inflammation signs?
5. What meaning does inflammation have for an organism?
Conclusion: ____________________________________________________________
______________________________________________________________________
______________________________________________________________________
______________________________________________________________________
8. Practice Examination.
Practice examination type 1: Choose the correct answer:
Test 1. Blood plasma of a healthy man contains several dozens of
proteins. During an illness new proteins can originate, namely the protein of
"acute phase". Select such protein from the listed below:
A. G immunoglobulin
B. Prothrombin
C. C-reactive protein
D. Fibrinogen
E. A immunoglobulin
Test 2. Which one of the following enzymes is the major component of
granulocytes producing antibacterial activity during acute inflammation?
A. Myeloperoxidase
B. Acid hydrolase
С. Protease
D. Lysozyme
Test 3. Humoral mediators of inflammation include:
А. Histamine
В. Interleukine
С. Prostaglandin Е2
D. Serotonin
Е. Bradykinin
Test 4. The first cells which migrate in area of inflammation are:
A. Neutrophiles
B. Lymphocytes
C. Monocytes
D. Eosinophiles
E. Basophiles
Test 5. Necrosis focus was observed in the area of hyperemia and skin
edema in a few hours after burn. What mechanism strengthens destructive
effects in the inflammation area?
A. Proliferation of fibroblasts
B. Diapedesis of erythrocytes
C. Primary alteration
D. Secondary alteration
E. Emigration of lymphocytes
Test 6. Prostaglandins (PGs) have effects on a variety of tissues. The
different prostaglandins may have different effects. Which of the following
is not correct statement?
A. The human arteriolar smooth muscle is relaxed by PgE2 and PgI2 where as
TxA2 and PgF2α cause vasoconstriction.
B. PgE2 has marked oxytocic action while PgF2α has tocolytic action.
Understanding Pathophysiology
Understanding Pathophysiology
Understanding Pathophysiology
Understanding Pathophysiology
Understanding Pathophysiology
Understanding Pathophysiology
Understanding Pathophysiology
Understanding Pathophysiology
Understanding Pathophysiology
Understanding Pathophysiology
Understanding Pathophysiology
Understanding Pathophysiology
Understanding Pathophysiology
Understanding Pathophysiology
Understanding Pathophysiology
Understanding Pathophysiology
Understanding Pathophysiology
Understanding Pathophysiology
Understanding Pathophysiology
Understanding Pathophysiology
Understanding Pathophysiology
Understanding Pathophysiology
Understanding Pathophysiology
Understanding Pathophysiology
Understanding Pathophysiology
Understanding Pathophysiology
Understanding Pathophysiology
Understanding Pathophysiology
Understanding Pathophysiology
Understanding Pathophysiology
Understanding Pathophysiology
Understanding Pathophysiology
Understanding Pathophysiology
Understanding Pathophysiology
Understanding Pathophysiology
Understanding Pathophysiology
Understanding Pathophysiology
Understanding Pathophysiology
Understanding Pathophysiology
Understanding Pathophysiology
Understanding Pathophysiology
Understanding Pathophysiology
Understanding Pathophysiology
Understanding Pathophysiology
Understanding Pathophysiology
Understanding Pathophysiology

More Related Content

What's hot

definition and scope of pathology
definition and scope of pathology definition and scope of pathology
definition and scope of pathology IMRANHOSSAIN323
 
Patho lec.unit 1. by maghan das
Patho lec.unit 1. by maghan dasPatho lec.unit 1. by maghan das
Patho lec.unit 1. by maghan dasMaghan Das
 
Illustrated veterinary pathology (General & systemic pathology)
Illustrated veterinary pathology (General & systemic pathology)Illustrated veterinary pathology (General & systemic pathology)
Illustrated veterinary pathology (General & systemic pathology)Dr. Waqas Nawaz
 
Pathophysiology1.
Pathophysiology1.Pathophysiology1.
Pathophysiology1.Shaikhani.
 
PathoPhysiology Chapter 1
PathoPhysiology Chapter 1PathoPhysiology Chapter 1
PathoPhysiology Chapter 1TheSlaps
 
Presentation for NGO " The Association for the Health of Society"
Presentation for NGO " The Association for the Health of Society"Presentation for NGO " The Association for the Health of Society"
Presentation for NGO " The Association for the Health of Society"Sofia glinyana
 
1 internal medicine
1 internal medicine1 internal medicine
1 internal medicineIam David
 
Diagnosing medical parasites
Diagnosing medical parasitesDiagnosing medical parasites
Diagnosing medical parasitesnizhgma.ru
 
General hygiene and medical ecology
General hygiene and medical ecologyGeneral hygiene and medical ecology
General hygiene and medical ecologyUmar Sahib
 
Paniker medical parasitology 8e
Paniker medical parasitology 8ePaniker medical parasitology 8e
Paniker medical parasitology 8eMohamedNabil730957
 
Introduction to Pathology
 Introduction to Pathology Introduction to Pathology
Introduction to PathologyBathsa Johnson
 

What's hot (20)

1 introduction
1 introduction1 introduction
1 introduction
 
definition and scope of pathology
definition and scope of pathology definition and scope of pathology
definition and scope of pathology
 
Patho lec.unit 1. by maghan das
Patho lec.unit 1. by maghan dasPatho lec.unit 1. by maghan das
Patho lec.unit 1. by maghan das
 
Illustrated veterinary pathology (General & systemic pathology)
Illustrated veterinary pathology (General & systemic pathology)Illustrated veterinary pathology (General & systemic pathology)
Illustrated veterinary pathology (General & systemic pathology)
 
Lecture 1 pathophysiology
Lecture 1 pathophysiologyLecture 1 pathophysiology
Lecture 1 pathophysiology
 
Pathophysiology1.
Pathophysiology1.Pathophysiology1.
Pathophysiology1.
 
Syllabus eng
Syllabus engSyllabus eng
Syllabus eng
 
PathoPhysiology Chapter 1
PathoPhysiology Chapter 1PathoPhysiology Chapter 1
PathoPhysiology Chapter 1
 
Presentation for NGO " The Association for the Health of Society"
Presentation for NGO " The Association for the Health of Society"Presentation for NGO " The Association for the Health of Society"
Presentation for NGO " The Association for the Health of Society"
 
Curso de medicina en dosis bajas aplicado en pediatría
Curso de medicina en dosis bajas aplicado en pediatríaCurso de medicina en dosis bajas aplicado en pediatría
Curso de medicina en dosis bajas aplicado en pediatría
 
1 internal medicine
1 internal medicine1 internal medicine
1 internal medicine
 
16 mlt
16 mlt16 mlt
16 mlt
 
Diagnosing medical parasites
Diagnosing medical parasitesDiagnosing medical parasites
Diagnosing medical parasites
 
General hygiene and medical ecology
General hygiene and medical ecologyGeneral hygiene and medical ecology
General hygiene and medical ecology
 
College of science
College of scienceCollege of science
College of science
 
Introduction
IntroductionIntroduction
Introduction
 
Paniker medical parasitology 8e
Paniker medical parasitology 8ePaniker medical parasitology 8e
Paniker medical parasitology 8e
 
NANOROBOT IN BIOMEDICIENE
NANOROBOT IN BIOMEDICIENENANOROBOT IN BIOMEDICIENE
NANOROBOT IN BIOMEDICIENE
 
Introduction to Pathology
 Introduction to Pathology Introduction to Pathology
Introduction to Pathology
 
Human experimentation
Human  experimentationHuman  experimentation
Human experimentation
 

Similar to Understanding Pathophysiology

PATHOLOGY INTRODUCTION.pptx
PATHOLOGY INTRODUCTION.pptxPATHOLOGY INTRODUCTION.pptx
PATHOLOGY INTRODUCTION.pptxsana usmani
 
04Introduction_to_Pathophysiology.pptx
04Introduction_to_Pathophysiology.pptx04Introduction_to_Pathophysiology.pptx
04Introduction_to_Pathophysiology.pptxSammyWeils
 
04Introduction_to_PF.ppt
04Introduction_to_PF.ppt04Introduction_to_PF.ppt
04Introduction_to_PF.pptAimanSiddiqui21
 
Lecture 1(introduction to pathology).
Lecture 1(introduction to pathology).Lecture 1(introduction to pathology).
Lecture 1(introduction to pathology).نصار ايوب
 
An introduction to experimental epidemiology
An introduction to experimental epidemiology An introduction to experimental epidemiology
An introduction to experimental epidemiology improvemed
 
Experimental epidemiology
 Experimental epidemiology Experimental epidemiology
Experimental epidemiologyimprovemed
 
EXPERIMENTAL EPIDEMIOLOGY
EXPERIMENTAL EPIDEMIOLOGYEXPERIMENTAL EPIDEMIOLOGY
EXPERIMENTAL EPIDEMIOLOGYimprovemed
 
INTRODUCTION TO PATHOLOGY (1).ppt
INTRODUCTION TO PATHOLOGY (1).pptINTRODUCTION TO PATHOLOGY (1).ppt
INTRODUCTION TO PATHOLOGY (1).pptKasongo8
 
Introduction to pathology
Introduction to pathology  Introduction to pathology
Introduction to pathology Dhiraj Shukla
 
Pathology
Pathology Pathology
Pathology ted_k
 
Introduction to pathology
Introduction to pathologyIntroduction to pathology
Introduction to pathologyVandana Thakur
 
Introduction to Pathology.pptx
Introduction to Pathology.pptxIntroduction to Pathology.pptx
Introduction to Pathology.pptxMohammedAbdela7
 
Maladaptive Responses to Immune DisordersMaladaptive responses to .docx
Maladaptive Responses to Immune DisordersMaladaptive responses to .docxMaladaptive Responses to Immune DisordersMaladaptive responses to .docx
Maladaptive Responses to Immune DisordersMaladaptive responses to .docxeubanksnefen
 
Introduction of pathology
Introduction of pathologyIntroduction of pathology
Introduction of pathologyNailaawal
 

Similar to Understanding Pathophysiology (20)

General teaching about disease
General teaching about diseaseGeneral teaching about disease
General teaching about disease
 
PATHOLOGY INTRODUCTION.pptx
PATHOLOGY INTRODUCTION.pptxPATHOLOGY INTRODUCTION.pptx
PATHOLOGY INTRODUCTION.pptx
 
04Introduction_to_Pathophysiology.pptx
04Introduction_to_Pathophysiology.pptx04Introduction_to_Pathophysiology.pptx
04Introduction_to_Pathophysiology.pptx
 
04Introduction_to_PF.ppt
04Introduction_to_PF.ppt04Introduction_to_PF.ppt
04Introduction_to_PF.ppt
 
Lecture 1(introduction to pathology).
Lecture 1(introduction to pathology).Lecture 1(introduction to pathology).
Lecture 1(introduction to pathology).
 
An introduction to experimental epidemiology
An introduction to experimental epidemiology An introduction to experimental epidemiology
An introduction to experimental epidemiology
 
Experimental epidemiology
 Experimental epidemiology Experimental epidemiology
Experimental epidemiology
 
EXPERIMENTAL EPIDEMIOLOGY
EXPERIMENTAL EPIDEMIOLOGYEXPERIMENTAL EPIDEMIOLOGY
EXPERIMENTAL EPIDEMIOLOGY
 
Pathology.ppt
Pathology.pptPathology.ppt
Pathology.ppt
 
Curso de medicina en dosis bajas aplicado en Pediatría
Curso de medicina en dosis bajas aplicado en PediatríaCurso de medicina en dosis bajas aplicado en Pediatría
Curso de medicina en dosis bajas aplicado en Pediatría
 
INTRODUCTION TO PATHOLOGY (1).ppt
INTRODUCTION TO PATHOLOGY (1).pptINTRODUCTION TO PATHOLOGY (1).ppt
INTRODUCTION TO PATHOLOGY (1).ppt
 
Introduction to pathology
Introduction to pathology  Introduction to pathology
Introduction to pathology
 
1. introduction
1. introduction1. introduction
1. introduction
 
Pathology
Pathology Pathology
Pathology
 
# 1 вступна
# 1 вступна# 1 вступна
# 1 вступна
 
Introduction to pathology
Introduction to pathologyIntroduction to pathology
Introduction to pathology
 
Branches of Medicine
Branches of MedicineBranches of Medicine
Branches of Medicine
 
Introduction to Pathology.pptx
Introduction to Pathology.pptxIntroduction to Pathology.pptx
Introduction to Pathology.pptx
 
Maladaptive Responses to Immune DisordersMaladaptive responses to .docx
Maladaptive Responses to Immune DisordersMaladaptive responses to .docxMaladaptive Responses to Immune DisordersMaladaptive responses to .docx
Maladaptive Responses to Immune DisordersMaladaptive responses to .docx
 
Introduction of pathology
Introduction of pathologyIntroduction of pathology
Introduction of pathology
 

More from Ivano-Frankivsk National Medical University

More from Ivano-Frankivsk National Medical University (20)

Ukrainian language lesson
Ukrainian language lessonUkrainian language lesson
Ukrainian language lesson
 
Pathophysiology of the liver 2018 (ukrainian language)
Pathophysiology of the liver 2018 (ukrainian language)Pathophysiology of the liver 2018 (ukrainian language)
Pathophysiology of the liver 2018 (ukrainian language)
 
Respiratory pathology
Respiratory pathologyRespiratory pathology
Respiratory pathology
 
Hypoxia
HypoxiaHypoxia
Hypoxia
 
Pathophysiology of cns 2017
Pathophysiology of cns  2017Pathophysiology of cns  2017
Pathophysiology of cns 2017
 
Endocrine 2016 all
Endocrine 2016 allEndocrine 2016 all
Endocrine 2016 all
 
Fever
FeverFever
Fever
 
Git stomatological faculty
Git stomatological facultyGit stomatological faculty
Git stomatological faculty
 
Cardiovascular pathology stomatological faculty
Cardiovascular pathology stomatological facultyCardiovascular pathology stomatological faculty
Cardiovascular pathology stomatological faculty
 
Allergy 2016
Allergy 2016Allergy 2016
Allergy 2016
 
Pathophysiology of extremal states
Pathophysiology of extremal statesPathophysiology of extremal states
Pathophysiology of extremal states
 
Anaemias
AnaemiasAnaemias
Anaemias
 
Pathophysiology of lipid metabolism
Pathophysiology of lipid metabolismPathophysiology of lipid metabolism
Pathophysiology of lipid metabolism
 
Pathophysiology of carbohydrates and proteins metabolism
Pathophysiology of carbohydrates and proteins metabolismPathophysiology of carbohydrates and proteins metabolism
Pathophysiology of carbohydrates and proteins metabolism
 
Tumors
TumorsTumors
Tumors
 
Inflammation med
Inflammation medInflammation med
Inflammation med
 
Typical disorders of peripheral blood flow
Typical disorders of peripheral blood flowTypical disorders of peripheral blood flow
Typical disorders of peripheral blood flow
 
Cell injury
Cell injuryCell injury
Cell injury
 
Pathophysiology of cns II 2013
Pathophysiology of cns II 2013Pathophysiology of cns II 2013
Pathophysiology of cns II 2013
 
Pathophysiology of cns
Pathophysiology of cns Pathophysiology of cns
Pathophysiology of cns
 

Recently uploaded

97111 47426 Call Girls In Delhi MUNIRKAA
97111 47426 Call Girls In Delhi MUNIRKAA97111 47426 Call Girls In Delhi MUNIRKAA
97111 47426 Call Girls In Delhi MUNIRKAAjennyeacort
 
Russian Call Girls Gunjur Mugalur Road : 7001305949 High Profile Model Escort...
Russian Call Girls Gunjur Mugalur Road : 7001305949 High Profile Model Escort...Russian Call Girls Gunjur Mugalur Road : 7001305949 High Profile Model Escort...
Russian Call Girls Gunjur Mugalur Road : 7001305949 High Profile Model Escort...narwatsonia7
 
Call Girls Jayanagar Just Call 7001305949 Top Class Call Girl Service Available
Call Girls Jayanagar Just Call 7001305949 Top Class Call Girl Service AvailableCall Girls Jayanagar Just Call 7001305949 Top Class Call Girl Service Available
Call Girls Jayanagar Just Call 7001305949 Top Class Call Girl Service Availablenarwatsonia7
 
See the 2,456 pharmacies on the National E-Pharmacy Platform
See the 2,456 pharmacies on the National E-Pharmacy PlatformSee the 2,456 pharmacies on the National E-Pharmacy Platform
See the 2,456 pharmacies on the National E-Pharmacy PlatformKweku Zurek
 
Call Girls Hsr Layout Just Call 7001305949 Top Class Call Girl Service Available
Call Girls Hsr Layout Just Call 7001305949 Top Class Call Girl Service AvailableCall Girls Hsr Layout Just Call 7001305949 Top Class Call Girl Service Available
Call Girls Hsr Layout Just Call 7001305949 Top Class Call Girl Service Availablenarwatsonia7
 
Call Girls Thane Just Call 9910780858 Get High Class Call Girls Service
Call Girls Thane Just Call 9910780858 Get High Class Call Girls ServiceCall Girls Thane Just Call 9910780858 Get High Class Call Girls Service
Call Girls Thane Just Call 9910780858 Get High Class Call Girls Servicesonalikaur4
 
Call Girl Service Bidadi - For 7001305949 Cheap & Best with original Photos
Call Girl Service Bidadi - For 7001305949 Cheap & Best with original PhotosCall Girl Service Bidadi - For 7001305949 Cheap & Best with original Photos
Call Girl Service Bidadi - For 7001305949 Cheap & Best with original Photosnarwatsonia7
 
Book Call Girls in Yelahanka - For 7001305949 Cheap & Best with original Photos
Book Call Girls in Yelahanka - For 7001305949 Cheap & Best with original PhotosBook Call Girls in Yelahanka - For 7001305949 Cheap & Best with original Photos
Book Call Girls in Yelahanka - For 7001305949 Cheap & Best with original Photosnarwatsonia7
 
Russian Call Girls Chickpet - 7001305949 Booking and charges genuine rate for...
Russian Call Girls Chickpet - 7001305949 Booking and charges genuine rate for...Russian Call Girls Chickpet - 7001305949 Booking and charges genuine rate for...
Russian Call Girls Chickpet - 7001305949 Booking and charges genuine rate for...narwatsonia7
 
Noida Sector 135 Call Girls ( 9873940964 ) Book Hot And Sexy Girls In A Few C...
Noida Sector 135 Call Girls ( 9873940964 ) Book Hot And Sexy Girls In A Few C...Noida Sector 135 Call Girls ( 9873940964 ) Book Hot And Sexy Girls In A Few C...
Noida Sector 135 Call Girls ( 9873940964 ) Book Hot And Sexy Girls In A Few C...rajnisinghkjn
 
Call Girl Surat Madhuri 7001305949 Independent Escort Service Surat
Call Girl Surat Madhuri 7001305949 Independent Escort Service SuratCall Girl Surat Madhuri 7001305949 Independent Escort Service Surat
Call Girl Surat Madhuri 7001305949 Independent Escort Service Suratnarwatsonia7
 
Asthma Review - GINA guidelines summary 2024
Asthma Review - GINA guidelines summary 2024Asthma Review - GINA guidelines summary 2024
Asthma Review - GINA guidelines summary 2024Gabriel Guevara MD
 
Call Girls Hebbal Just Call 7001305949 Top Class Call Girl Service Available
Call Girls Hebbal Just Call 7001305949 Top Class Call Girl Service AvailableCall Girls Hebbal Just Call 7001305949 Top Class Call Girl Service Available
Call Girls Hebbal Just Call 7001305949 Top Class Call Girl Service Availablenarwatsonia7
 
Call Girls Hosur Just Call 7001305949 Top Class Call Girl Service Available
Call Girls Hosur Just Call 7001305949 Top Class Call Girl Service AvailableCall Girls Hosur Just Call 7001305949 Top Class Call Girl Service Available
Call Girls Hosur Just Call 7001305949 Top Class Call Girl Service Availablenarwatsonia7
 
Call Girls Service Noida Maya 9711199012 Independent Escort Service Noida
Call Girls Service Noida Maya 9711199012 Independent Escort Service NoidaCall Girls Service Noida Maya 9711199012 Independent Escort Service Noida
Call Girls Service Noida Maya 9711199012 Independent Escort Service NoidaPooja Gupta
 
call girls in green park DELHI 🔝 >༒9540349809 🔝 genuine Escort Service 🔝✔️✔️
call girls in green park  DELHI 🔝 >༒9540349809 🔝 genuine Escort Service 🔝✔️✔️call girls in green park  DELHI 🔝 >༒9540349809 🔝 genuine Escort Service 🔝✔️✔️
call girls in green park DELHI 🔝 >༒9540349809 🔝 genuine Escort Service 🔝✔️✔️saminamagar
 
Low Rate Call Girls Mumbai Suman 9910780858 Independent Escort Service Mumbai
Low Rate Call Girls Mumbai Suman 9910780858 Independent Escort Service MumbaiLow Rate Call Girls Mumbai Suman 9910780858 Independent Escort Service Mumbai
Low Rate Call Girls Mumbai Suman 9910780858 Independent Escort Service Mumbaisonalikaur4
 
Hematology and Immunology - Leukocytes Functions
Hematology and Immunology - Leukocytes FunctionsHematology and Immunology - Leukocytes Functions
Hematology and Immunology - Leukocytes FunctionsMedicoseAcademics
 
Call Girls In Andheri East Call 9920874524 Book Hot And Sexy Girls
Call Girls In Andheri East Call 9920874524 Book Hot And Sexy GirlsCall Girls In Andheri East Call 9920874524 Book Hot And Sexy Girls
Call Girls In Andheri East Call 9920874524 Book Hot And Sexy Girlsnehamumbai
 
Housewife Call Girls Hsr Layout - Call 7001305949 Rs-3500 with A/C Room Cash ...
Housewife Call Girls Hsr Layout - Call 7001305949 Rs-3500 with A/C Room Cash ...Housewife Call Girls Hsr Layout - Call 7001305949 Rs-3500 with A/C Room Cash ...
Housewife Call Girls Hsr Layout - Call 7001305949 Rs-3500 with A/C Room Cash ...narwatsonia7
 

Recently uploaded (20)

97111 47426 Call Girls In Delhi MUNIRKAA
97111 47426 Call Girls In Delhi MUNIRKAA97111 47426 Call Girls In Delhi MUNIRKAA
97111 47426 Call Girls In Delhi MUNIRKAA
 
Russian Call Girls Gunjur Mugalur Road : 7001305949 High Profile Model Escort...
Russian Call Girls Gunjur Mugalur Road : 7001305949 High Profile Model Escort...Russian Call Girls Gunjur Mugalur Road : 7001305949 High Profile Model Escort...
Russian Call Girls Gunjur Mugalur Road : 7001305949 High Profile Model Escort...
 
Call Girls Jayanagar Just Call 7001305949 Top Class Call Girl Service Available
Call Girls Jayanagar Just Call 7001305949 Top Class Call Girl Service AvailableCall Girls Jayanagar Just Call 7001305949 Top Class Call Girl Service Available
Call Girls Jayanagar Just Call 7001305949 Top Class Call Girl Service Available
 
See the 2,456 pharmacies on the National E-Pharmacy Platform
See the 2,456 pharmacies on the National E-Pharmacy PlatformSee the 2,456 pharmacies on the National E-Pharmacy Platform
See the 2,456 pharmacies on the National E-Pharmacy Platform
 
Call Girls Hsr Layout Just Call 7001305949 Top Class Call Girl Service Available
Call Girls Hsr Layout Just Call 7001305949 Top Class Call Girl Service AvailableCall Girls Hsr Layout Just Call 7001305949 Top Class Call Girl Service Available
Call Girls Hsr Layout Just Call 7001305949 Top Class Call Girl Service Available
 
Call Girls Thane Just Call 9910780858 Get High Class Call Girls Service
Call Girls Thane Just Call 9910780858 Get High Class Call Girls ServiceCall Girls Thane Just Call 9910780858 Get High Class Call Girls Service
Call Girls Thane Just Call 9910780858 Get High Class Call Girls Service
 
Call Girl Service Bidadi - For 7001305949 Cheap & Best with original Photos
Call Girl Service Bidadi - For 7001305949 Cheap & Best with original PhotosCall Girl Service Bidadi - For 7001305949 Cheap & Best with original Photos
Call Girl Service Bidadi - For 7001305949 Cheap & Best with original Photos
 
Book Call Girls in Yelahanka - For 7001305949 Cheap & Best with original Photos
Book Call Girls in Yelahanka - For 7001305949 Cheap & Best with original PhotosBook Call Girls in Yelahanka - For 7001305949 Cheap & Best with original Photos
Book Call Girls in Yelahanka - For 7001305949 Cheap & Best with original Photos
 
Russian Call Girls Chickpet - 7001305949 Booking and charges genuine rate for...
Russian Call Girls Chickpet - 7001305949 Booking and charges genuine rate for...Russian Call Girls Chickpet - 7001305949 Booking and charges genuine rate for...
Russian Call Girls Chickpet - 7001305949 Booking and charges genuine rate for...
 
Noida Sector 135 Call Girls ( 9873940964 ) Book Hot And Sexy Girls In A Few C...
Noida Sector 135 Call Girls ( 9873940964 ) Book Hot And Sexy Girls In A Few C...Noida Sector 135 Call Girls ( 9873940964 ) Book Hot And Sexy Girls In A Few C...
Noida Sector 135 Call Girls ( 9873940964 ) Book Hot And Sexy Girls In A Few C...
 
Call Girl Surat Madhuri 7001305949 Independent Escort Service Surat
Call Girl Surat Madhuri 7001305949 Independent Escort Service SuratCall Girl Surat Madhuri 7001305949 Independent Escort Service Surat
Call Girl Surat Madhuri 7001305949 Independent Escort Service Surat
 
Asthma Review - GINA guidelines summary 2024
Asthma Review - GINA guidelines summary 2024Asthma Review - GINA guidelines summary 2024
Asthma Review - GINA guidelines summary 2024
 
Call Girls Hebbal Just Call 7001305949 Top Class Call Girl Service Available
Call Girls Hebbal Just Call 7001305949 Top Class Call Girl Service AvailableCall Girls Hebbal Just Call 7001305949 Top Class Call Girl Service Available
Call Girls Hebbal Just Call 7001305949 Top Class Call Girl Service Available
 
Call Girls Hosur Just Call 7001305949 Top Class Call Girl Service Available
Call Girls Hosur Just Call 7001305949 Top Class Call Girl Service AvailableCall Girls Hosur Just Call 7001305949 Top Class Call Girl Service Available
Call Girls Hosur Just Call 7001305949 Top Class Call Girl Service Available
 
Call Girls Service Noida Maya 9711199012 Independent Escort Service Noida
Call Girls Service Noida Maya 9711199012 Independent Escort Service NoidaCall Girls Service Noida Maya 9711199012 Independent Escort Service Noida
Call Girls Service Noida Maya 9711199012 Independent Escort Service Noida
 
call girls in green park DELHI 🔝 >༒9540349809 🔝 genuine Escort Service 🔝✔️✔️
call girls in green park  DELHI 🔝 >༒9540349809 🔝 genuine Escort Service 🔝✔️✔️call girls in green park  DELHI 🔝 >༒9540349809 🔝 genuine Escort Service 🔝✔️✔️
call girls in green park DELHI 🔝 >༒9540349809 🔝 genuine Escort Service 🔝✔️✔️
 
Low Rate Call Girls Mumbai Suman 9910780858 Independent Escort Service Mumbai
Low Rate Call Girls Mumbai Suman 9910780858 Independent Escort Service MumbaiLow Rate Call Girls Mumbai Suman 9910780858 Independent Escort Service Mumbai
Low Rate Call Girls Mumbai Suman 9910780858 Independent Escort Service Mumbai
 
Hematology and Immunology - Leukocytes Functions
Hematology and Immunology - Leukocytes FunctionsHematology and Immunology - Leukocytes Functions
Hematology and Immunology - Leukocytes Functions
 
Call Girls In Andheri East Call 9920874524 Book Hot And Sexy Girls
Call Girls In Andheri East Call 9920874524 Book Hot And Sexy GirlsCall Girls In Andheri East Call 9920874524 Book Hot And Sexy Girls
Call Girls In Andheri East Call 9920874524 Book Hot And Sexy Girls
 
Housewife Call Girls Hsr Layout - Call 7001305949 Rs-3500 with A/C Room Cash ...
Housewife Call Girls Hsr Layout - Call 7001305949 Rs-3500 with A/C Room Cash ...Housewife Call Girls Hsr Layout - Call 7001305949 Rs-3500 with A/C Room Cash ...
Housewife Call Girls Hsr Layout - Call 7001305949 Rs-3500 with A/C Room Cash ...
 

Understanding Pathophysiology

  • 1. Ministry of Public Health Service of Ukraine Ivano-Frankivsk National Medical University Pathophysiology MODULE 1 GENERAL PATHOLOGY Training-methodical manual for class and out-of-class work for medical faculty students Prepared by: Gerasymchuk M. R. Cherkasova V. V. Zaiats L. M. Ivano-Frankivsk, 2017
  • 2. 2 «The general nosology. Typical pathological processes» Training-methodical manual for class and out-of-class work for medical students / M.R. Gerasymchuk, V.V. Cherkasova, L.M. Zaiats // IFNMU. Department of pathological physiology. – 2017. – 92 p. Discussed and approved on the profile commission of medical&biological disciplines meeting of Ivano-Frankivsk National Medical University. Protocol № __ from «__» _______ 2017 year
  • 3. 3 CALENDAR PLAN OF PRACTICAL STUDIES of pathological physiology for the students of the III course in the V semester THEME OF PRACTICAL STUDIES DATES 1. The object, and tasks of pathophysiology. Methods of pathophysiological investigations. Basic steps in pathological physiology development. 04.09–08.09 2. General nosology. The doctrine of etiology and pathogenesis. Pathogenic effect of ionizing radiation on the body. 11.09–15.09 3. The role of heredity and constitution in pathology. 18.09–22.09 4. Pathology of reactivity. Violation of immunological reactivity. 25.09–29.09 5. Hypersensitivity reactions of immediate and delayed type. 02.10–06.10 6. Cell pathophysiology. 09.10–13.10 7. Typical disorders of peripheral blood flow and microcirculation. 16.10–20.10 8. Inflammation: phlogogenic factors, pathogenesis of alteration, mediators. Kongeim’s experiment. 23.10–27.10 9. Inflammation: pathogenesis of exudation and proliferation. 30.10–03.11 10. Disorders of thermal metabolism. Fever. 06.11–10.11 11. Tumors. 13.11–17.11 12. Starvation.Hypoxia. 20.11–24.11 13. Disorders of carbohydrate metabolism. Diabetes mellitus. 27.11–01.12 14. Disorders of lipid metabolism. Medical and social problems of obesity. 04.12–08.12 15. Disorders of water-electrolyte metabolism and microelements metabolism. 11.12–15.12 16. Module 1 (practical part) 18.12–22.12 17. Module 2 (theoretical part) 25.12–29.12 The ESTIMATION FOR THE MODULE is defined as a sum of marks of current educational activity (in points), which is proposed during the evaluation of theoretical knowledges and practical skills. Maximal amount of points, which a student can collect - 200 points during of every module study, including for current educational activity – 120 points (together module topics are 120 points), on results final module control are 80 points. Control of theoretical and practical preparation 0 – 2 points – completely prepared homework; 0 – 5 points – oral answer; 0 – 1 points – test control during class. Minimum – 0 points; positive – 4; maximum – 8 points
  • 4. 4 Topic № 1. The object, and tasks of pathophysiology. Methods of pathophysiological investigations. Basic steps in pathological physiology development. General nosology. Teaching about etiology and pathogenesis. Pathogenic effect of radiation and electric current. 1. The The actuality of the theme. Pathological physiology is science, that studies the functional changes at a sick man and animals. It studies the most general conformities to the law of origin, development, consequences of illness. By experiment, we may reproduce and study on animals the separate models of illnesses, violation of organs and systems for the cognition of basic conformities to the law of development of illnesses of man. Consequently, the experiment is the basic method of pathophysiology. 2. Duration of the class – 1 hour 30 min. 3. Aim: To know such terms as the «modelling», «experiment». To be able: a) The modeling of various forms of pathologic processes, protective and adaptive reactions of humans; b) Experimental therapy as an important method of studying and introducing the new ways of treatment; c) Clinical studying of various diseases with functional, biochemical, immunological and other tests due to pathophysiology groundation therapy. To perform practical work: 1. Main features and purpose of experiment in pathophysiology. Correlation of method of clinical supervision with the pathophysiological experiment. 2. Possible experiment on a human? Mental and ethical aspects of pathophysiological experiment. 3. Causes and consequences constantly change their places. The cause (etiological factors) causes the pathologic reactions (process) and then these reactions return to the first agent (etiological factor) and intensify it. 4. To analyse the “vicious circle” in the pathogenesis. Causes and consequences constantly change their places. 4. Basic level. The name of the previous and future disciplines The receiving of the skills 1. histology 2. biochemistry 3. physiology 4. internal medicine 5. reanimatology 6. radiology Ionizing radiation. Main characteristics. Physical and chemical ionizing radiation interchange in organism. Gas composition of air and partial pressure of oxygen. Functional units structure of respiratory system, blood, blood circulation system. 5. Control questions of the theme: 1. Term definition – what is pathophysiology? Explain role in medicine.
  • 5. 5 2. History of pathophysiology development. Significance of scientific works of K. Bernar, R. Virchow, U. Kongeim, I. I. Mechnikov, G. Selie and other famous investigators. The origin of pathological physiology as a scientific discipline. Formation and development of pathological physiology in Ukraine. Scientific schools of pathophysiologists, the main directions of their activity. 3. The connect between the pathophysiology and other disciplines. General pathophysiology, special or systemic and clinical pathophysiology. 4. Methods of pathological physiology. An experiment as the main method of pathophysiology, its significance for solving fundamental problems of medicine. 5. Name the methods and the aims of pathophysiology. Gist of experimental method. Modeling pathological processes and diseases on animals: possibilities and limitations. Significance of comparative-evolutional method for development of pathophysiology. 6. Rules of work with laboratory animals. Types of experiment. General principles of planning the experimental investigations, count, statistic processing and analysis of the results. 7. Definition a notion “experiment”, peculiarities, significance and lacks of acute experiment, peculiarities of chronic experiment. Experimental therapy. 8. General doctrine of disease. The basic concepts of general nosology. Norm, health, disease (definition of WHO). A pathological process. A pathological condition, pathological reaction. 9. Disease as a biological, medical and social problem. The abstract and the concrete in the concept “disease”. Unity of the destructive and protective in disease. Principles of disease classification, classification of WHO. The main laws of a disease course. 10.Stages of disease development. Remission, relapse, complication. Variants of disease outcome: complete and incomplete recovery. 11.Terminal conditions: pre-agony, agony, clinical death. Pathophysiological bases of reanimation. 12. The main directions in the development of doctrine of disease: humoral (Hippocrates), cellular (R. Virchow). Their development in the modern stage. 13. Definition of the concept “etiology”. Problem of causality in pathology. The role of causes and conditions in occurrence of diseases. The main directions in the development of doctrine of etiology: monocausalism, conditionalism, constitutionalism, psychosomatic conception, etc. 14.Modern concepts of causality in pathology. Classification of etiological factors. External and internal etiological factors. The concept of risk factors. “Diseases of civilization”. Ecological, genetic, accumulative and ontogenetic conceptions of occurrence of human diseases. Etiotropic principles of prevention and treatment.
  • 6. 6 15.Definition of the concept “pathogenesis”. The destructive and adaptive phenomena of pathogenesis. Manifestations of injury at different levels: a molecular, cellular, tissue, organ, organism one. 16. Protective reactions of adaptation. Adaptation, compensation. Mechanisms of immediate and long-term adaptation. The role of nervous and humoral factors in their realization. “Compensatory-adaptative reactions”, types of compensatory-adaptative reactions, role of compensatory-adaptative reactions in convalescence mechanisms. 17. The cause-effect relations in pathogenesis. Variants of direct cause- effect relations. “Circulars vicious”. The main link of pathogenesis. Role of the local and general in pathogenesis. 18. Concept of localization and generalization of pathological processes. Ways of their distribution. Specific and nonspecific mechanisms of pathogenesis. Pathogenetic principles of disease treatment. 19.Pathogenic effect of ionizing radiation. Types of ionizing radiation. Radiosensitivity of tissues. Mechanisms of direct and indirect radiation damage of biological structures. Water radiolysis. Radiotoxins. 20.Manifestations of radiation damages on molecular, cellular, tissue, organ and system levels. 21.Pathogenesis of radiation sickness, its main forms and syndromes. Early and late effects of large and small doses of ionizing radiation. 22.Natural mechanisms of antiradiation protection. Pathophysiological bases of radioprotection. 23.Influence of low atmosphere pressure upon organism, man activity in condition of low atmosphere pressure, mechanisms of low atmosphere pressure action, manifestations. 24.Influence of high atmosphere pressure upon organism, man activity in condition of high atmosphere pressure, mechanism of high atmosphere pressure action, manifestations of saturation and desaturation, hyperbaric oxygenation use in medicine. 25.Pathogenic influence of electric current. 6. Independent audience work of student. Protocol № 1 Date_____________________ Experimental work 1. Determining the amount of haemoglobin. Solution of hydrochloric acid pours in a test-tube of hemometer to the number 2 on the scale. Then collects 0.02 ml blood in capillary and outpour it in a test tube. Mixture leaves for 5 minutes. After distilled water pour full in the test-tube until the color of liquid in a test tube will be evened with the color of standard solution. Calculate the amount of haemoglobin in mmol/l. Formula of
  • 7. 7 calculation: B • 0,6206, where B – is an amount of haemoglobin in g%; 0,6206 – is a coefficient of count in unit of SI. Conclusion:____________________________________________________________ Experimental work 2. Determining the amount of erythrocytes. In a test- tube pour 4 ml of a 3% solution of chloride of sodium. By a capillary pipette collect 0,02 ml blood and produce it on the bottom of test tube. The contents is carefully mixed. Then drop of liquids by pipette place under preliminary grinding (rubbing) in integumentary (covered) small glass of account chamber. Count up erythrocytes in 5 large (that in 80 small) squares of net of Goryaeva and calculate their amount in 1 litr of blood after a formula: lТ ААА / 100 10 100 10 80 2004000 128   where: A – is an amount of red corpuscles in 5 large squares; 4000 – the volume of small square makes 1/4000 mm3; 200 – is dilution of blood; 80 – is an amount of the counted up small squares; 108 is a multiplier for the count of amount of red corpuscles in unit of SI; T – 1012. Conclusion:____________________________________________________________ Experimental work 3. Count of amount of leucocytes. In a test-tube pour 0,4 ml of a 3% solution of vinegar acid. By a capillary pipette collect 0,02 ml blood and outpour it on the bottom of test-tube. Mixture is mixed, and then fills the chamber of Goryaeva. Count up the amount of leucocytes in 100 large squares of net. Expect the amount of leucocytes in 1 l of blood. lG АА Х / 20 10 11600 204000 6     , where: A – is an amount of leucocytes in 100 large squares; 1600 – is an amount of the calculated small squares; 4000 1 it is a volume of small square in mm3; 20 – is a degree dilution of blood; 106 – is a multiplier for the count of amount of leucocytes in unit of SI; G – giga - 109. Conclusion:____________________________________________________________ 8. Practice Examination. Practice examination type 1. Choose the correct answer: Test 1. Which scientist emphasized senescence of connective tissue cells cytoplasm?
  • 8. A. Bogomolets B. Mechnikov C. Dilman D. Frolkis E. Berdichev Test 2. The people, living in endemic centre, has recovered the three-day malaria. In one and a half year after moving in the other places occured malaria once again. What the most probable form of this disease? A. Superinfection B. Reinfection C. The Relapse D. Persistent infection E. Secondary infection Test 3. Which one of the methods is the most important at pathophysiology? A. Epidemiological B. Anatomical C. Clinical D. Experimental Test 4. Through day after irradiation (the dose 3,5 Gr.) beside damaged in shelters: erythrocytes-4,7 1012 /l, hemoglobin-9,0 mmol/l, leukocytes-11 109 /l, thrombocytes-270 109 /l. In daub shelters: basophils-1%, eosinophils- 3%, neutrophils-81%, lymphocytes-11%, monocytes-4%. For what period sharp radiation sickness typical specified shifts? A. The Primary reaction B. The Peak disease C. The Rumpled welfare D. Reconstruction E. Terminations Test 5. Sudden death during dehermitizing of aircraft at height 19000 m is rose owing to: A. Carbonic acid deficit B. Organism desaturation C. Gas embolism D. Oxygen deficit Е. Height meteorism Test 6. A 49-year-old man was suffering 12 years ago from rheumatic myocarditis, endocarditis, and insufficiency of mitral valve. Examinations showed the absence of inflammatory process, sufficient minute blood volume. What is it? A. Pathological reaction B. Pathological process C. Typical pathological process D. Compensatory reaction E. Pathological condition Test 7. A 39-years-old patient has been suffering from gastric ulcer for last 4 years. Pain in epigastric region, heartburn, nausea, and constipation appear mainly in autumn and spring. Name this condition. A. Remission B. Acute period C. Complication D. Pathologic condition E. Relapse Test 8. Gasping respiration appears in a patient with severe lung pathology. What terminal condition is this characteristic for? A. Agony B. Pre-agony
  • 9. 9 C. Clinical death D. Biological death E. Terminal pause Test 9. Respiratory standstill developed in a man as a result of action of electric current from town mains for 0.1 seconds with position of electrodes “right hand – head”. Indicate the most probable reason for this complication. A. Paralysis of inspiration center B. Emotional stress C. Paralysis of respiratory muscles D. Reflex respiratory standstill (pain shock) E. Total paralysis of respiratory center Practice examination type 2 Give answer to the questions of the real-life tasks: Task 1. Experimentator should study mechanismas of dyspnea in case of disorders of blood circulation in the lungs. Which main periods of pathophysiological experiment he should make? 1 3 2 4 Task 2. In dog under anesthesia, was destroied cusp of mitral valve and in this ways reproduce cardiac insufficiency. Which method of disease modeling used? ______________________________________________________________________ Task 3. As a result of damage one of atomic electric station blocks arose emissions of ionizing rays products. In zone of raised radiation activity found oneself three men. Approximately they got on 250-300 Rad. They immediately delivered in clinic. 1. What consequences do follow to wait in victim? 2. What periods pick out in development of radial sick? 3. What researches you should make for clearing up of lesion degree? 4. How would be differ state irradiated being received 50R? 500R? 6000R? Answerfor the task: ____________________________________________________ __________________________________________________________________________ __________________________________________________________________________ __________________________________________________________________________ __________________________________________________________________________ __________________________________________________________________________ Practice examination type 3 I. Give the description of each form of radiation disease (1 – acute, 2 – chronic):
  • 10. 10 Indicator 1/2 Indicator 1/2 A The disturbance of hemo- poiesis and blood system (lymphopenia, thrombocytopenia) F Hemorrhagic syndrome G Sexual dysfunction B Anemia H Spasm paralytic syndrome C Immune reactivity decrease J Shock D Dysfunction of the alimentary tract, vomiting, anorexia, diarrhea K Asthenia E Necrotic tonsillitis L Trophic disorders II. For each statement choose T (true) or F (false) in the list provided. № Statement T F 1 The first period of acute radiation disease (ARD) with duration from several hours to 1-2 days is characterized by excitation, headache, instability of the vegetative functions, lability of the arterial pressure and pulse 2 Latent period (one week) is accompanied by leukopenia (progressing of lymphocytopenia, development of granulocytopenia). 3 The third period is characterized by progressing leukopenia, anemia. Hemorrhagic syndrome develops. Decrease of immunologic reactivity. 4 The outcome of the ARD is multiple inflammatory processes (necrotic angina, pneumonia, frontitis and others). III. Give answer to the questions of the real-life task: TASKS 1. What direction of electricity through the human body is the most dangerous? A Head C Heart B Kidney D Liver 2. When the man is less sensitive to the electric current? A Under narcosis C Hypoxia B Tiredness D Alcohol intoxication 3. The most resistant to electric current are all the below mentioned except: A External epidermal layer D Muscles B Tendons and bones E Blood C Nerves F Cerebrospinal fluid Signature___________________
  • 11. 11 Literature: Basic: 1. Robbins and Cotran Pathologic Basis of Disease 9thed./Kumar, Abbas, Fauto.–2015.–Ch.9.–P.426–432. 2. General and clinical pathophysiology. Edited by prof. A.V. Kubyskin. Simferopol. – 2011. – P. 17–85. 3. Pathophysiology, N.K. Symeonova. Kyiv, AUS medicine Publishing. – 2010. – P. 24–34. 4. Copstead Lee-Ellen C. Pathophysiology / Lee-Ellen C. Copstead, Jacquelyn L. Banasic // Elsevier Inc. – 2010. – P. 3–13, 1280–1283. Additional: 1. Pathophysiology, Concepts of Altered Health States, Carol Mattson Porth, Glenn Matfin.– New York, Milwaukee. – 2009. – P. 2–10. 2. J.B.Walter I.C.Talbot General pathology. Seventh edition. – 1996. – P. 1–17. Topic 2: Role of heredity, constitution and aging changes in pathology. 1. The actuality of the theme. Genetic and congenital defects are important at all levels of healthcare because they affect all age groups and can involve almost any of the body tissues and organs. Congenital defects, sometimes called birth defects, develop during prenatal life and usually are apparent at birth or shortly thereafter. About 2000 different hereditary diseases are known now. 4 % of new-born suffer from these or other genetically conditioned defects. Heredity pathology plays an important role in development of such hereditary conditioned diseases, as atherosclerosis, essential hypertension, rheumatism, diabetes mellitus, and gout. 2. Duration of the class – 1 h 30 min. 3. Aim: To know the role of hereditary factors and constitution in the development of diseases and pathological processes. To be able: to analyze the pathogenesis of chromosomal diseases, phenylketonuria. To perform practical work: Characterize fragile X-associated mental retardation syndrome produces a unique combination of phenotypic features that affect the central nervous system, the testes, and the cranial skeleton. 4. Basic level. The name of the previous and future disciplines The receiving of the skills 1. histology 2. biochemistry 3. physiology 4. pediatrics Encoding of hereditary information. Legitimacy and types of hereditary signs transmission in generations. Cytological genetic base. 5. Control questions of the theme: 1.Heredity as a cause and condition of disease development. Relation of the hereditary and acquired in pathogenesis. Hereditary and congenital diseases. Gene- and phenocopies. Classification of hereditary diseases.
  • 12. 12 2.Mutations. Principles of their classification. Causes of mutations. Mutagenic factors of a physical, chemical and biological origin. 3.Mendelian disorders: diseases caused by single-gene defects: • autosomal dominant (Huntington disease, neurofibromatosis type 1, Marfan syndrome, Familial hypercholesterolemia (LDL receptor deficiency), acute intermittent porphyria); • autosomal recessive (Cystic fibrosis, sickle cell anemia, phenylketonuria, Tay-Sachs disease (hexosaminidase A deficiency)); • X-linked (Fragile X syndrome, hypophosphatemic rickets, Duchenne muscular dystrophy, Lesch-Nyhan syndrome, Glucose-6-phosphate dehydrogenase deficiency, Hemophilia A and B, red-green color blindness, Menke’s disease). 4.Monogenic hereditary diseases. Antigen associative diseases. 4a. Diseases caused by mutations in genes encoding structural proteins: Marfan Syndrome, Ehlers-Danlos Syndromes 4b. Diseases Caused by Mutations in Genes Encoding Receptor Proteins or Channels: Familial Hypercholesterolemia, Cystic Fibrosis; 4c. Diseases Caused by Mutations in Genes Encoding Enzyme Proteins: phenylketonuria, Galactosemia; Lysosomal Storage Diseases: Tay-Sachs Disease (GM2 Gangliosidosis: Deficiency in Hexosaminidase β Subunit), Niemann-Pick Disease Types A, B and C, Gaucher Disease, Mucopolysaccharidoses; Glycogen Storage Diseases (Glycogenoses): von Gierke disease, Pompe disease, McArdle disease; 4d. Mitochondrial diseases: Leber hereditary optic neuropathy, MELAS: mitochondrial encephalomyopathy, lactic acidosis, and stroke-like episodes), myoclonic epilepsy with ragged red muscle fibers). 4e. Diseases caused by alterations of imprinted regions: Prader-Willi and Angelman Syndromes. 5.Systems of anti mutational protection. Mechanisms of DNA reparation. The role of disorders of reparative systems and “immune control” in an occurrence of hereditary pathology. 6.Chromosomal diseases. Mechanisms of occurrence of the genome and chromosomal mutations. Polyploidy, aneuploidy, deletion, duplication, inversion, translocation. 6a. Cytogenetic disorders involving autosomes: trisomy 21 (Down Syndrome), 22q11.2 deletion syndrome, trisomy 18 (Edwards syndrome), trisomy 13 (Patau syndrome); 6b. Cytogenetic disorders involving sex chromosomes: Klinefelter syndrome, Turner syndrome, “Super-woman” syndrome. 7.Congenital anomalies: polydactyly, cleft lip, cleft palate. Etiology. Pathogenesis. Perinatal infections (TORCH Toxoplasma (T), rubella virus (R),
  • 13. 13 cytomegalovirus (C), herpesvirus (H), and any of a number of other (O) microbes such as Treponema pallidum). 8.Methods of investigation, prophylaxis, and treatment of hereditary diseases. Ways of correction of genetic defects. Outlooks of gene engineering. 9.Constitution, its role in pathology. Classification of constitutional types by Hippocrates, Sigot, Kretchmer, I. P. Pavlov, A. A. Bogomolets. The concept of diatheses. 6. Students’ practical activities. Protocol № 2 Date_____________________ Experimental work 1. There is research of sexual chromatin (Bara's bodies) in the epithelium cells of mucous layer of mouth. The impression smear (touch smear) prepares from the mucous layer of mouth. The drop of a 1% solution of acetoorcein on the smear. The drop is covered integumentary glass so that paint evenly spread on the stroke and air under glass must be absent. A surplus of paint is taken off by a filtration paper. Study preparation under the small increase of microscope (lens×20, eyepiece×10). Find the accumulation of cells and study it under immersion increase (lens×90, eyepiece×10). Interphase nuclei are counted and mark how many they have contained the Bara's bodies. Count up 100 cells. For an analysis, it is necessary to take away unharmed cells with a round or oval nucleus. Sexual chromatin is placed under a nuclear membrane, has a semilunar or triangle shape. For women – 28% all nuclei of cells of epitheliums contain sexual chromatin, and for men - 0-1%. Conclusion:____________________________________________________________ Experimental work 2. To define content of phenylpyruvic acid in urine. For the exposure of phenylpyruvic acid in urine use the test-paper of type “Biofan”, which are imbued by buffer solution of chlorous iron. The end of test- paper is put into urine and through 30 sec. the results. A test is considered positive if the test-paper becomes a green color. Conclusion:____________________________________________________________ 7. Practice Examination. Practice examination type 1. Choose the correct answer: Test 1. Which of the following characteristics are most typical of multifactorial inheritance? A. Sex predilection B. Mitochondrial inheritance C. Recurrence risks reflect the number of affected relatives
  • 14. 14 D. Major cause of miscarriages E. Maternally derived Test 2. In which variant of karyotype in the nucleus of somatic cells only one Bar’s body can be found? A. 47 XX, 15+ B. 45 XO C. 45 XY D.47 XY, 21+ E. 48 XXXY Test 3. At examining of a patient-doctor stressed attention to her low growth, wide shieldlike, insufficient physical and sexual development. External genitalia formed on a feminine type, internal was underdeveloped. Barr body in cells of mouth mucous membrane was not determined. For what disease is it typical? A. Klinefelter’s syndrome B. Turner’s syndrome C. Х-trisomy syndrome D. Down’s illness Е. Pathau’s syndrome Test 4. Which of the following is most likely in an untreated child with phenylketonuria (PKU)? A. Elevated tyrosine B. Increased skin pigmentation C. Normal phenylalanine hydroxylase levels D. Elevated alanine E. Decreased skin pigmentation Test 5. The boy was born by woman 45 years with fission of the upper lip ("cleft lip" and "cleft palate"). Significant breaches are discovered. Under additional examination on the part of nervous, cardiovascular systems and visions. At study, Karyotype is diagnosed trisomy on 13 chromosomes. What syndrome exists beside boy? A. Klinefelter B. Edwards C. Turner D. Patau E. Down Practice examination type 2. Give answers to the questions of the picture and real-life task: A young woman is referred for genetic counseling. She has a 3-year-old boy with developmental delay and small joint hyperextensibility. The pediatrician has diagnosed fragile X-associated mental retardation. She is currently pregnant with her second child at 14 weeks of gestation. The family history is unremarkable. 1. What is the genetic mutation responsible for fragile X-associated mental retardation? How does it cause the clinical syndrome of developmental delay, joint hyperextensibility, large testes, and facial abnormalities? 2. Which parent is the probable carrier of the genetic mutation? Explain why this parent and the grandparents are phenotypically unaffected. 3. What is the likelihood that the unborn child will be affected? Answer for the task: ____________________________________________ ______________________________________________________________________ ______________________________________________________________________
  • 16. 16 Practice examination type 4 For each statement write T (true) or F (false) in the blank provided № Statement T F 1 Hypertension is a multifactorial disease. 2 Fat intake is a risk factor for many diseases. 3 A multifactorial trait is expressed when multiple genes and environmental influences blend together. 4 In Down’s syndrome, the pathology is manifested independently of an environment. 5 It is easy to distinguish between the effects of shared environmental factors and the effects of common pool genes. 6 Psychiatric disorders do not manifest familial patterns. 7 Relative risk is a ratio between incidence and individuals. 8 Early type II diabetes may develop when an individual’s diet changes to heavy carbohydrate consumption. 9 Finding and understanding environmental factors that affect penetrance of specific genes is important if chronic familial diseases are to be prevented. 10 A variation in the phenotype for different genotypes caused by the environmental factors is a threshold liability trait. 11 The frequency of genetic disease in the population depends on phenotypes. 12 Risk factors, when removed or eliminated, delay or prevent disease. 13 A proband is the individual beginning the pedigree. 14 The recurrence risk is less when more than one sibling is affected. 15 The existence of a particular risk factor indicates an individual will develop a specific disease. 16 The expression of a disease may require both an inherited defect and environmental exposure. 17 Multifactorial diseases can change substantially from one population to another because gene frequencies and environments differ. 18 Dizygotic twins are identical. 19 Recurrence risk is higher if the disease is more severe in the proband. 20 The prevalence rate is the number of individuals affected by a disease. 21 The incidence rate is the number of persons who have died from the disease.
  • 17. 17 Signature___________________ Literature: Basic: 1. Robbins and Cotran Pathologic Basis of Disease 9th international ed./ V.Kumar, A.K.Abbas, J.C.Aster – 2015. – Chapters 5, 10. – P. 137-182, 451–480. 2. Robbins and Cotran Pathologic Basis of Disease 9thed./ Kumar, Abbas, Fauto. – 2013.–Ch6.–P. 215–268. 3. General and clinical pathophysiology. Ed. by prof. A.V. Kubyskin. Simferopol. – 2011. – P. 86–104. 4. Copstead Lee-Ellen C. Pathophysiology / Lee-Ellen C.Copstead, J.L.Banasic//Else.Inc.–2010.–P.86–127. 5. Essentials of Pathophysiology: Concepts of Altered Health States (Lippincott Williams & Wilkins), Trade paperback (2003) / Carol Mattson Porth, Kathryn J. Gaspard. Chapters 3-4– P. 36 – 62. 6. Symeonova N.K. Pathophysiology. – N.K. Symeonova. Kyiv, AUS M-ne Publishing. – 2010. – P. 44–65. 7. Pathophysiology, Concepts of Altered Health States, C.M.Porth, G. Matfin.– NY,M. –2009.–P.133–156. Additional: 8. Gozhenko A.I. General and clinical pathophysiology / A.I. Gozhenko, I.P. Gurcalova // Study guide for medical students and practitioners. Edited by prof.Zaporozan, OSMU. – Odessa. – 2005.– P. 49–60. 9. Silbernagl S. Color Atlas of Pathophysiology / S. Silbernagl, F. Lang // Thieme.St. NY. – 2000. – P. 2–9. 10. J.B.Walter I.C.Talbot General pathology. Seventh edition. – 1996. – P. 59–87. Topic 3. Pathology of reactivity. Disorders of immune reactivity. AIDS. 1. The actuality of the theme. Reactivity is the characteristic of the organism to react in a certain way on the influence of the environment. It is the same as growing up, feeding, and metabolism. Any pathological process in one or another degree changes the reactivity of the organism and in the time the changing reactivity which exceeds physiological of the organism can become a main development of the disease. 2. Duration of the class – 1h 30min. 3. Aim: to know types of reactivity. To be able: to analyze the specific and non-specific reactivity. To perform practical work: Resistance. Types. Interaction with reactivity. 4. Basic level. The name of the previous and future disciplines The receiving of the skills 1. histology 2. biochemistry 3. physiology 4. immunology 5. internal medicine Structure of the blood-brain barrier and others hystohaematic barriers. Physiological reactivity indexes. Structure of central nervous system, eye, thyroid gland, internal ear, and testis. 5. The advice for students . Properties of Human Immunoglobulins Class Ig Content in blood serum mg/L % of total Ig level Mole- cular mass kD Fixatio n of comple- ment Trans- ference hrough the placent Function IgG gen. 8-16 80 150 ++ + Secondary immune response, protection from bacteria and viruses displays antiviral,IgG1 65 150 ++ +
  • 18. 18 6. Control questions of the theme: 1.Definition of the concept of reactivity. Reactivity as a condition of disease development. Types of reactivity. 2.Dependence of reactivity on sex, age, heredity, condition of immune, nervous and endocrine systems. Influence of environment on the reactivity of organism. Manifestations of reactivity on a molecular, cellular, tissue, organ, system level and on the level of the whole organism. 3.Reactivity by Bogomolets. Explain term “physiological system of connecting tissue” and name elements of this system by Bogomoletz. What is a role of this system in the maintenance of organisms’ homeostasis? 4.The theory of Hans’a Seley of diseases of adaptation. 5.Reactivity and biological barriers. 6.Definition of the concept of resistance. Passive and active resistance. The connection of resistance with reactivity. IgG2 23 150 + + antitoxin, and antibacterial properties; only Ig that crosses the placenta; responsible for protection of newborn; activates complement and binds to macrophages IgG3 8 150 +++ + IgG4 4 150 - + IgM 0.5-2 6 900 +++ - Primary immune response Forms the natural antibodies such as those for ABO blood antigens; prominent in early immune responses; activates complement IgA plasm. 1.4-1.9 13 160 - - Predominant Ig in bodysecretions, such as saliva, nasal and respiratory secretions, and breast milk; protects mucous membranes IgA secr. 2-5 380 - - Mucous membrane protection IgD 0-0.4 0-1 180 - - Membrane receptors found on B lymphocytes; needed for maturation of B cells IgE 14-450 ng/ml 0.002 190 - - Reagines, protection from parasites. Binds to mast cells and basophils; involved in parasitic infections, allergic and hypersensitivity reactions.
  • 19. 19 7.Mechanisms of nonspecific resistance. Biological barriers, their classification, significance for a resistance of organism. 8.Humoral factors of nonspecific resistance of an organism to infectious agents (lysozyme, C-reactive protein, interferons). 9.Complement system and its disorders. 10.Phagocytosis, its mechanisms. Disorders of phagocytosis: causes, mechanisms, consequences. Chediak-Higashi syndrome. 11.Mechanisms of the normal immune response of a humoral and cellular type, their disorders. Role of lymphocytes, macrophages, eosinophils, neutrophils. Major Histocompatibility Complex Molecules: The peptide display system of adaptive immunity. 12.Immunologic tolerance: central and peripheral. 13.Immunodeficiency, a definition of the concept, classification (WHO). Bruton’s X-linked agammaglobulinemia, Common variable immunodeficiency, Ig A deficiency, X-linked hyper-IgM Syndrome, DiGeorge’s syndrome, Severe Combined Immunodeficiency, agammaglobulinemia (Swiss type), Wiscott- Aldrich syndrome. 14.Pathogenesis of the main clinical manifestations of isolated and combined disorders of functions of T- and B-lymphocytes. Causes, mechanisms of development and types of initial immunodeficiencies. The role of physical, chemical and biological factors in the development of secondary immunodeficiencies (clinical examples). 15. Pathophysiological characteristics of the acquired immunodeficiency syndrome (AIDS). 7. Students’ practical activities. Protocol № 3 Date_____________________ Experimental work 1. To learn the violations of barriers function and adaptations of an organism in the conditions of a convulsive attack. A test it is carried out on two mise or young rats. On employment 1% solution of trypan blue subcutaneously in animals, from a calculation 2 ml on 100 gr of mass. In 30 min 0,3 ml enter intraperitoneal camphorated oil to one of the animals. This animal is placed under a glass hubcap and mark time of offensive of cramps, their character and time of death. Animals are killed and studied organs, a substance of brain and liquid eye. Conclusion: ____________________________________________________________ ______________________________________________________________________ ______________________________________________________________________ ______________________________________________________________________
  • 20. 20 Experimental work 2. An action of decreased atmospheric pressure upon organism of a rat (experiment). Place animal under a glass bell. Acquaint with a condition of the animal before experiment beginning (behavior, skin color, breathing rate, a reaction to sound irritant). By the medium of pump gradually diminish pressure under the bell. Observe change of animal condition. Clock each supervision. After the beginning of first cramps in animal gradually let air in and observes renewal of organism functions. Write down the experiment results in the table and protocol of experiment according to the scheme: Animal Beginning of experiment Behavior Breath rate Color of visible cutaneous covering Time of offensive of cramps 1.Adult 2.New born Conclusion: ____________________________________________________________ ______________________________________________________________________ ______________________________________________________________________ ______________________________________________________________________ 8. Practice Examination. Practice examination type 1. Choose the correct answer: Test 1. The deficient content of immunoglobulins was revealed in a patient. What cells of immune system produce immunoglobulins? A. Plasma cells B. T-killers C. B-lymphocytes D. T-helpers E. NK-killers Test 2. The gene coding for HLA antigen is present on: A. 6 q B. 6 p C. 16 p D. 16 q E. 21 p Test 3. The complement component with opsonin activity is: А. C1 В. С3а C. C3b D. С5а Test 4. Various cells of the oral mucous membrane and antimicrobial substances synthesized by these cells play an important part in the local immunity of the oral cavity. Specify the key factors for the local immunity: A. Eosinophils B. B-lymphocytes
  • 21. 21 C. IgG D. Macrophages E. Secretory IgA Test 5. Which cytokine promotes the proliferation of T and B lymphocytes? A. IFN-γ (interferon γ) B. IL-2 (interleukin 2) C. IL-4 (interleukin 4) D. TNF-α (tumor necrosis factor α) E.TGF-β (transforming growth factor β) Test 6. The immunoblot detected gp120 protein in the blood serum. This protein is typical for the following disease: A. Virus B hepatitis B. HIV-infection C. Tuberculosis D. Syphilis E. Poliomyelitis Test 7. Blood analysis of a patient showed signs of HIV infection (human immunodeficiency virus). Which cells does HIV-virus primarily? A. Specialized nervous cells (neurons) B. Mast cells C. Cells that contain receptor IgM (B-lymphocytes) D. Proliferating cells (stem hematoplastic cells) E. Cells that contain receptor T4 (T-helpers) Test 8. A pregnant woman was detected to have IgM to rubella virus. An obstetriciangynecologist recommended therapeutic abortion due to the high risk of teratogenic affection of the fetus. Detection of IgM was of great importance as it is these specific immunoglobulins that: A. Indicate recent infection B. Penetrate placental barrier C. Have the largest molecular weight D. Are associated with anaphylactic reactions E. Are the main factor of antiviral protection Test 9. A 5-year-old child is diagnosed with Bruton syndrome (X-linked agammaglobulinemia) that manifests itself in severe clinical course of bacterial infections and absence of B lymphocytes and plasma cells. What changes of immunoglobulin content can be observed in blood serum of the child with immunodeficiency? A. Increased IgA, IgM B. Decreased IgA, IgM C. Decreased IgD, IgE D. Increased IgD, IgE E. No changes Test 10. A 13-year-old boy presents with eczematous rashes on his shins and torso. Anamnesis states cases of otitis, pneumonia, and furuncles in the patient. Blood test: platelets - 70 · 109 /l, low activity of T helper and T
  • 22. 22 suppressor cells, low IgM, with normal IgA and IgG. What immunodeficient disease does this boy have? A. DiGeorge syndrome B. Louis-Bar syndrome (Ataxiatelangiectasia) C. Severe combined immunodeficiency (Swiss type) D. Wiskott-Aldrich syndrome E. Chediak-Higashi syndrome Practice examination type 2. Give answers to the questions of the real- life tasks: Task 1. An 18-year-old man presents with complaints of fever, facial pain, and nasal congestion consistent with a diagnosis of acute sinusitis. His medical history is notable for multiple sinus infections, two episodes of pneumonia, and chronic diarrhea, all suggestive of primary immunodeficiency syndrome. Workup establishes a diagnosis of common variable immunodeficiency. 1.What are the common infectious manifestations of common variable immunodeficiency? 2. What are the underlying immunologic abnormalities responsible for these infectious manifestations? 3. What other diseases is this patient at increased risk for? 4. What treatment is indicated? Answerfor the task 1: ___________________________________________________ __________________________________________________________________________ __________________________________________________________________________ __________________________________________________________________________ __________________________________________________________________________ __________________________________________________________________________ __________________________________________________________________________ Task 2. The patient was hospitalized with complaints on cough, rise of temperature up to 38-39 C, weakness, and headache. Illness was related to super cooling. Pneumonia was diagnosed after clinical research. 1. What was a direct cause of the disease? 2. What role in disease beginning did cooling play? 3. What reactivity mechanisms did lead to organism resistance reduce? 4. What are your recommendations for organism resistance rising to catarrhal diseases? Answer for the task 2: ___________________________________________ __________________________________________________________________________ __________________________________________________________________________ __________________________________________________________________________ __________________________________________________________________________ __________________________________________________________________________ __________________________________________________________________________ Signature___________________ Literature: Basic: 1. Robbins and Cotran Pathologic Basis of Disease 9th international edition./ V.Kumar, A.K.Abbas,
  • 23. 23 J.C.Aster – 2015. – Chapter 6. – P. 186–200, 211–231, 237–263. 2. Robbins Basic Pathology 9th edition./ Kumar, Abbas, Fauto. – 2013. – Chapter 4. – P. 99–158. 3. General and clinical pathophysiology. Ed. by prof. A.V. Kubyskin. Simferopol. – 2011. – P. 209–233. 4. Pathophysiology, Concepts of Altered Health States, C.M.Porth, G.Matfin.– NY. – 2009. – P. 400–427. 5. Symeonova N.K. Pathophysiology. – N.K. Symeonova. Kyiv, AUS M-ne Publ. – 2010. – P. 65–84. 6. Copstead Lee-Ellen C. Pathophysiology / Lee-Ellen C. Copstead, Jacquelyn L. Banasic // Elsevier Inc. – 2010. – P. 203–221, 263–286. 7. Essentials of Pathophysiology: Concepts of Altered Health States (Lippincott Williams & Wilkins), Trade paperback (2003) / Carol Mattson Porth, Kathryn J. Gaspard. Ch.7-8, 10. – P. 121 – 149, 178–189. Additional: 1. Robbins and Cotran Pathologic Basis of Disease 8th edition./ Kumar, Abbas, Fauto. – 2007. – Chapter 5. – P. 108–118, 152–165. 2. Silbernagl S. Color Atlas of Pathophysiology / S. Silbernagl, F. Lang // Thieme.. NY. – 2000. – P. 42–47. 3. Gozhenko A.I. General and clinical pathophysiology / A.I. Gozhenko, I.P. Gurcalova // Study guide for medical students and practitioners. Ed. by prof. Zaporozan, OSMU. – Odessa. – 2005.– P. 42–48. Topic 4. Immediate types of hypersensitivity reactions. 1. The actuality of the theme. The immediate-type allergy is often met in practical activity of physicians of various specialties. That is because of the great environmental pollution by industrial products, chemical matters, and allergens of vegetable animal, bacterial, fungus origin and also due to a wide use of various drugs. That type of allergy can develop suddenly. The severity of their proceeding is various – from slight reactions to anaphylactic shock dangerous for one’s life. Preventing and treatment of the allergy reactions is based on knowledge about their development mechanisms. 2. Duration of the class – 1 h 30 min. 3. Aim: To know that allergy is a complex of breaches, appearing in our organism by the humoral immunological reactions. Both inflammation and allergy is a protective response on the exo- and endogenous factor. To be able: to analyze the pathogenesis of allergy by A.D. Ado. To perform practical work: to analyze the mechanisms of immunologically mediated disorders by Coombs and Gell (1968) 2. 4. Basic level. The name of the previous and future disciplines The receiving of the skills 1. Histology 2. Biochemistry 3. Physiology 4. Immunology 5. Anaestesiology Structure and function of organs and vessels. Immunity and its mechanisms. Antigens and antibodies, their structure and function. Sensitizing. 5. The advice for students . Classificaton of allergic reactions by Coombs and Gell (1968) № Type Prototype disorder Immune mechanism 1. Anaphylactic type Anaphylaxia, some forms of bronchial asthma Formation of IgE (cytotropic) antibody → release of vasoactive
  • 24. 24 amines and other mediators from basophils and mast cells. 2. Cytotoxic type Autoimmune hemolytic anemia, erythroblastosis feta- lis, cytotoxic reactions the action of large doses ofBogo- mglets’s ACS (antireticular cytotoxic serum) Formation of IgG, IgM → binds to antigen on target cell surface → phagocytosis of target cell or lysis by C8,9 3. Immune complex disease Arthus reaction serum sickness, systemic lupus erythematosus, certain forms of acute glomerulonephritis Ag + Ab →activated Co → attracted complexes neutrophills → release of lysosomal enzymes 4. Cell – mediated (delayer) hypersensivity Tuberculosis, contact dermatitis, transplant rejection Sensitized thymus-derived T-lymphocytes → release of lymphokines and T-cell-mediated cytotoxicity. 5. By Roight Stimulating allergic reactions. Via BAS, hormones, mediators Collagen diseases, connective tissue diseases, rheumatoid arthritis and other. The cells, containing Ag, begin to function intensively under the influence of Ab. Than cells are secreted hormones or mediators. 6. Control questions of the theme: 1. Allergy. Definition of the concept and general characteristics of allergy. 2. Allergy and immunity. Etiology of allergy, kinds of exo- and endogenous allergens. The significance of hereditary and acquired factors in the development of allergy. 3. Principles of classification of allergic reactions. General characteristics of allergic reactions by Cumbs and Gell. Stages of pathogenesis of allergic reactions. 4. Anaphylactic reactions: experimental models, main clinical forms. Immune mechanisms of anaphylactic reactions, the role of mast cells in their development. Active and passive anaphylaxis, pathogenesis of anaphylactic shock. 5. Cytotoxic reactions: experimental modeling, main clinical forms. Mechanisms of antibody-mediated diseases. Mechanisms of cytolysis. Examples of diseases, cytotoxic: autoimmune hemolytic anemia, acute rheumatic fever, Goodpasture syndrome, transfusion reaction, autoimmune thrombocytopenic purpura; non-cytotoxic: Graves disease (hyperthyroidism), myasthenia gravis, insulin-resistant diabetes, pernicious anemia. 6. Immune complex-mediated reactions, experimental modeling, pathogenesis, clinical forms. Factors determining pathogenesis of immune complexes. Immune complex damages, their local and general manifestations. Examples of diseases: Systemic lupus erythematosus, rheumatoid arthritis,
  • 25. 25 poststreptococcal glomerulonephritis, serum sickness, Arthus reaction, Sjögren syndrome. 7. Students’ practical activities. Protocol № 4 Date_____________________ Experimental work. Anaphylactic shock in a guinea pig. Watching a movie by the results of experiment and analysis of observed results. Reproduction: 0.5 ml of horse serum was injected into a peritoneal cavity of a guinea pig two weeks before the main experiment. In a day of experiment guinea pig is fixed on a desk, then researcher cutes skin along medial line of the neck, and separates jugular vein. After this, he injects 2.0 ml of the horse serum into the jugular vein and students observe anaphylactic shock manifestation. After animal death researcher separates cardiac-pulmonary complex. Pay attention to the character of damage in the lungs. It is necessary to answer on questions at prepare of protocol in part «Discussion». 1) What type of allergy is it? Explain. 2) What is the mechanism of animal sensitization? 3) What is the role of biologically active substances in anaphylactic shock manifestation? 4) What was the reason for animal death? Explain why. 5) Is it possible to cause an anaphylactic shock by repeat injection of horse serum if an experiment could be unsuccessful? Explain. Through 1.5- 2 min. mark the first symptoms of shock: scratching of snout, standing of wool, an anxiety of animal, cough, cyanosis of a snout. A next symptom is departing of excrement and urine. Cramps, at first tonic (a guinea pig falls on a side, quotation marks pronate, muscles are tense), and then clonic (shallow cramps of extremities), liquid breathing, with large pauses. Death comes in default of breathing and work of the heart is stored. Determination of degree of shock: - I stage (+) - characterized by standing of wool; - II stage (++) is standing of wool, cough, scratching of snout; - III stage (+++) --standing of wool, cough, scratching of snout, departing of excrement and urine, cramp; - IV stage (++++) -- standing of wool, cough, scratching of snout, departing of excrement and urine, death. It is convenient to analyze the dynamics of anaphylactic-type of allergy using, as an illustration, the experimental parenteral injection of a heteroserum to a healthy animal (twice with the two-week interval). Conclusion: ____________________________________________________________ __________________________________________________________________________ __________________________________________________________________________ __________________________________________________________________________
  • 26. 26 8. Practice Examination. Practice examination type 1. Choose the correct answer: Test 1. Anaphylactic shock developed in a patient with botulism after second injection of antitoxic antibotulinus serum mixture. What is the main mechanism of anaphylaxis? A. Interaction of T-lymphocytes with mediators B. Interaction of antigen with IgM C. Interaction of macrophages with antigens D. Interaction of antigen with IgE E. Interaction of T-lymphocytes with tissue basophils Test 2. In the 1960s, it was quickly ascertained that Peace Corpus workers sent to schistosome-endemic areas were exposed to massive initial doses of cercariae before any protective immunity existed. In these individuals, IgG antibodies developed in response to the developing worms, and when the adults began their prodigious release of eggs into the circulation, the patients suffered acute and potentially life-threatening symptoms of fever, edema, arthralgia, and rush. Which of the following is another condition that arises by a similar immunologic mechanism? A. Atopic allergy B. Arthus reaction C. Goodpasture syndrome D. Tuberculin reaction E. Transfusion reaction Test 3. Catarrhal inflammation of bulbar conjunctiva and nose mucous membrane develop in patient every year in spring and early summer, when trees and flowers are in blossom. Production of specific antibodies to pollen underlies this syndrome. What cells activate and develop exocytosis in this syndrome? A. Neutrophils B. Macrophages C. Lymphocytes D. Mast cells E. Throbocytes Test 4. It is known that bronchial asthma develops by mechanism of immediate hypersensitivity, which includes 3 sequential stages: A. Immunological, pathochemical, pathophisiological B. Pathochemical, pathophisiological, immunological C. Pathochemical, immunological, pathophisiological D. Pathophisiological, immunological, pathochemical E. Pathophisiological, pathochemical, immunological Test 5. Man with the caries is subjected to constant sensitization by streptococcus antigen. What disease can appear due to this etiological factor? A. Glomerulonephritis B. Pancreatitis C. Myocarditis D. Pulpits E. Periodontitis
  • 27. Test 6. A child with diphtheria 10 days after injection of antitoxic antidiphtherial serum has developed skin rash, accompanied by severe itch, rising temperature up to 380C and joints pain. What is the cause of these symptoms? A. Delayed type of hypersensitivity B. Anaphylactic reaction C. Contact allergy D. Atopia E. Serum sickness Practice examination type 2. Give answer to the questions of the real-life task: The 36 year old worker was hospitalized to dermatological department of clinic. He complains on rash skin hands and itch. The rash appeared two months ago. An application skin test with nickel sulphate was positive. A test on of macrophages migration inhibition with nickel is positive too. 1. What one does testify allergy nature of the disease? 2. Is there any base to recognize this disease as a delayed allergy? Answerfor the task______________________________________________________ __________________________________________________________________________ __________________________________________________________________________ __________________________________________________________________________ __________________________________________________________________________ Practice examination type 3. Put number of these illustrations (1 to 6) in the correct order as to how anaphylaxis develops with brief explanation of each step.
  • 28. 28 Signature___________________ Literature: Basic: 1. Robbins and Cotran Pathologic Basis of Disease 9thed./Kumar, Abbas, Fauto.–2013.– Ch.4.–P.109–117. 2. General and clinical pathophysiology. Edited by prof. A.V. Kubyskin. Simfer. – 2011. – P. 233–257. 3. Copstead Lee-Ellen C. Pathophysiology / Lee-Ellen C. Copstead, Jacquelyn L. Banasic // Elsevier Inc. – 2010. – P. 222–241. 4. Pathophysiology, Concepts of Altered Health States, Carol Mattson Porth, Glenn Matfin.– New York, Milwaukee. – 2009. – P. 410–424. 5. Symeonova N.K. Pathophysiology. – N.K. Symeonova. Kyiv, AUS medicine Publ. – 2010.–P.87–100. Additional: 1. Robbins and Cotran Pathologic Basis of Disease 8thed./Kumar, Abbas, Fauto. –2007.–Ch.7.–P.119–152. 2. Gozhenko A.I. General and clinical pathophysiology / A.I. Gozhenko, I.P. Gurcalova // Study guide for medical students and practitioners. Edited by prof. Zaporozan, OSMU. – Odessa. – 2005.– P. 96–104. 3. Essentials of Pathophysiology: Concepts of Altered Health States (Lippincott Williams & Wilkins), Trade paperback (2003) / Carol Mattson Porth, Kathryn J. Gaspard. Chapter 10. – P. 168 – 177. 4. Silbernagl S. Color Atlas of Pathophysiology / S. Silbernagl, F. Lang // Thieme. Stuttgart. New York. – 2000. – P. 52–59. Topic 5: Reactions of hypersensitivity of a delayed type. Autoimune diseases. Transplantology. 1. The actuality of the theme. The human immune network is a multifaceted defense system that has evolved to protect against invading microorganisms, prevent the proliferation of cancer cells, and mediate the healing of damaged tissue. Under normal conditions, the immune response deters or prevents disease. However, occasionally the inadequate, inappropriate, or misdirected activation of the immune system can lead to debilitating or life- threatening illnesses, typified by allergic or hypersensitivity reactions, transplantation immunopathology, autoimmune disorders, and immunodeficiency states. Preventing and treatment of the allergy reactions is based on knowledge about their development mechanisms. 2. Duration of the class – 1 h 30 min. 3. Aim: To know that allergy is a complex of breaches, appearing in our organism by the cellular immunological reactions. Both inflammation and allergy is protective response on the exo- and endogenous factor. To be able: 1) determine the leading link of the cellular immune reactions pathogenesis and cause-effect relations of separate pathogenetic mechanisms; 2) draw a conclusion about the character of pathological changes during cellular immune reactions in organism and possible mechanisms of their development on the ground of functional and biochemical investigation data. To perform practical work: to analyse the mechanisms of cellular immunologycally mediated disorders by Coombs and Gell (1968) and explain stimulating allergic reactions by Roight.
  • 29. 29 By Roight – stimulating allergic reactions. Via BAS, hormones, mediators the cells, containing Ag, begin to function intensively under the influence of Ab. Than cells are secreted hormones or mediators. Ex. Collagen diseases, connective tissue diseases, rheumatoid arthritis and other. 4. Basic level. The name of the previous and future disciplines The receiving of the skills 1. histology 2. biochemistry 3. physiology 4. immunology Structure and function of organs and vessels. Influence of BAS on vessel wall. Cellular Immunity and its mechanisms. Antigens and antibodies, their structure and function. 5. Control questions of the theme: 1. Reactions of hypersensitivity of a delayed type: experimental modeling, main clinical forms. Features of immune mechanisms. The role of lymphokines. 2. Autoallergic diseases. Causes and mechanisms of their development. The role of autoallergic component in pathogenesis of diseases. 3. Pseudoallergic reactions. 4. The main principles of prophylaxis and treatment of allergic reactions. Desensibilization. 5. The experimental modeling of pathology of the immune system. 6. Pathophysiological bases of transplantation of organs and tissues. 7. Immune tolerance, its types. Methods of experimental modeling of immune tolerance. 8. Mechanisms of development of immune tolerance and their disorders. 9. Immune interrelations in system “mother-fetus”. 10.“Graft versus host” reaction, conditions of its development, acute and chronic forms. 11.“Host versus graft” reaction, conditions of its development, acute and chronic forms. 12.The main principles of immune stimulation and immune suppression. 13.V types of allergic reactions. 6. Students’ practical activities. Protocol № 5 Date_____________________ Experiment 1. To learn displays and analyze the mechanism of development of hypersensitiveness of delayed type into a rat. For the receipt of hypersensitiveness of slow type animal, three days prior to experience sensitize (introduction to the pillow of paw tuberculin Freund's adjuvant in an amount 100 MCL. 3 times with an interval in two weeks. Before 24 hours to experiment enter intraperitoneally) anaphylaxis-provoking dose of
  • 30. 30 tuberculin (0.3 ml). On session for a rat under anesthesia open an abdominal region, look after the displays of allergic reaction from the side peritoneum and prepare strokes-imprints. Strokes dye after Romanovsky's stain. The dried upstroke is fixed three minutes in the glass with a methyl alcohol, and then dyes Romanovsky's stain during 20-25 min. The morphological displays of hypersensitiveness of slow type study under the immersion increase of microscope. In the strokes-imprints of experimental animals find plenty of lymphocytes and monocytes. Comparison the strokes of animals of controls paint out also. The results of experience describe and sketch. To explain displays and mechanism of development of hypersensitiveness of slow type into a rat on the basis of findings facts. Conclusion: ____________________________________________________________ __________________________________________________________________________ __________________________________________________________________________ 7. Practice Examination. Practice examination type 1. Choose the correct answer: Test 1. Allergic diagnostic tests are used for the diagnosis of many infectious diseases (tuberculosis, brucelosis, tularemia etc). Diagnosis is confirmed if papula and redness appear in the place of the allergen injection. Antigens interaction reaction is conditioned by: A. IgE and lymphokines B. IgM and macrophages C. T-lymphocytes and lymphokines D. IgE and T-lymphocytes E. IgM and tissue basophiles Test 2. Thyrotoxicosis was diagnosed in a patient. Antithyroid antibodies were found in his blood. Which type of allergic reaction is observed at development of this disease? A. Immune complex-mediated B. Stimulating C. Anaphylactic D. Cytotoxic E. Delayed type hypersensitivity Test 3. Mycobacterium tuberculosis results in an intracellular bacterial infection that provokes which one of the following immune responses? A. Natural killer cytotoxic response B. CD8-positive cytotoxic T cell response C. Complement mediated lysis of infected cell D. T helper 1 delayed type hypersensitivity response E. Eosinophilia Test 4. Hyperergic inflammation form of upper respiratory tract (larynx, trachea, bronchi) develops at a 6-year-old child. Threat of the respiratory impairment develops and then necessity of using anti-
  • 31. 31 inflammatory hormones occurs. Which hormone has anti-inflammatory property? A. Cortisol B. Adrenaline C. Growth hormone D. Testosterone E. Insulin Test 5. An alloimmune disorder is: A. Graft rejection B. Insulin-dependent diabetes C. Myxedema D. All of the above are correct E. None of the above is correct. Practice examination type 2. Give answers to the questions of the real- life tasks: Task 1. Equal sizes skin allotransplant was engrafted to two groups of rabbits. A herewith leucocytes suspension taken from rabbits-donors of allotransplant was infected intravenously to one group rabbit 2 weeks before transplantation. In what group of rabbits rejection of skin transplants will occur first? Why? Answerfor the task 1_____________________________________________________ ______________________________________________________________________________________ __________________________________________________________________________ Task 2. Skin allotransplant 6 dm2 was engrafted to of rats. Rejection occurred during first 14 days. 30 days after rejection of allotransplant the same donor’s skin were engrafted to same animals. What will be life duration of transplants after repeated engrafting? Explain, why? Answerfor the task 3_____________________________________________________ __________________________________________________________________________ __________________________________________________________________________ Task 3. The 36 year old worker was hospitalized to dermatological department of clinic. He complains on rash skin hands and itch. The rash appeared two months ago. An application skin test with nickel sulphate was positive. A test on of macrophages migration inhibition with nickel is positive too. 1. What one does testify allergy nature of the disease? 2. Is there any base to recognize this disease as a delayed allergy? Answerfor the task 4_____________________________________________________ __________________________________________________________________________ __________________________________________________________________________ Practice examination type 3: Rebus riddle. Solve the riddle to learn a characteristic of a skin disorder.
  • 32. 32 __________________________________________________________________________ __________________________________________________________________________ Signature___________________ Literature: Basic: 1. Robbins and Cotran Pathologic Basis of Disease 9thed./Kumar, Abbas, Fauto.–2013.–Ch.4.–P.122–139. 2. General and clinical pathophysiology. Edited by prof. A.V. Kubyskin. Simf. – 2011. – P. 233–249. 3. Symeonova N.K. Pathophysiology. – N.K. Symeonova. Kyiv, AUS medicine Publ. – 2010.–P.95-100. 4. Copstead Lee-Ellen C. Pathophysiology / Lee-Ellen C. Copstead, Jacquelyn L. Banasic // Elsevier Inc. – 2010. – P. 222–241. 5. Pathophysiology, Concepts of Altered Health States, Carol Mattson Porth, Glenn Matfin.– New York, Milwaukee. – 2009. – P. 416–424. Additional: 1. Robbins and Cotran Pathologic Basis of Disease8thed./Kumar, Abbas, Fauto.–2007–Ch.7.–P.119–131. 2. Gozhenko A.I. General and clinical pathophysiology / A.I. Gozhenko, I.P. Gurcalova // Study guide for medical students and practitioners. Edited by prof. Zaporozan, OSMU. – Odessa. – 2005.– P. 96–104. 3. Essentials of Pathophysiology: Concepts of Altered Health States (Lippincott Williams & Wilkins), Trade paperback (2003) / Carol Mattson Porth, Kathryn J. Gaspard. Chapters 10– P. 168 – 172. 4. Silbernagl S. Color Atlas of Pathophysiology / S. Silbernagl, F. Lang // Thieme. NY. – 2000. – P. 52–55. Topic 6. Cell injury. General mechanisms of cell damage. 1. The actuality of the theme. Cellular injury can be caused by any factor that disrupts cellular structures or deprives the cell of oxygen and nutrients required for survival. The injury may be reversible (sublethal) or irreversible (lethal) and is classified broadly as chemical, hypoxia (lack of sufficient oxygen), free radical, or infectious. Cellular injuries from various causes have different clinical and pathophysiologic manifestations.
  • 33. 33 Cellular death is confirmed by structural changes seen when cells are stained and examined under a microscope. No biochemical indicators of cellular death are universally applicable because we still do not know precisely what biochemical functions must be compromised before a cell dies.2. Duration of the class – 1h 30min. 3. Aim: to know the cell responses to injury and the major types of cellular necrosis. To be able: to analyze the main molecular mechanism of cell membrane damage, cellular adaptations occurring in atrophy, hypertrophy, hyperplasia, dysplasia, metaplasia and two pathways of apoptosis. To perform practical work: describe the cell responses on injury and mechanisms of apoptosis. 4. Basic level. The name of the previous and future disciplines The receiving of the skills 1. histology 2. biochemistry 3. physiology 4. pathomorphology 5. surgery Types and sources of radial energy. Properties of ionizing rays. Use of X-rays and radioactive elements in popular equipment and medicine. Main measures safety due to deal with X rays and radioactive particles. Ecological catastrophes with radioactive environment pollution. Blood cells and methods of their count. A bioelectrical activity of central nervous system and its registration. Fluid mosaic model of the plasma membrane. Cell populations and cycle landmarks. Lipid peroxidation. Erythrocytes in hypertonic, isotonic, and hypotonic solutions. 1. The advice for students. Structural-functional organization of a cell: The nucleus contains genetic material of a cell. The membrane provides the integrity of cell structures. Lysosomes contain a wide spectrum of hydrolytic enzymes. Mitochondria provide energy needs of a cell by synthesis of ATP. Ribosomes carry out protein synthesis. The endoplasmic reticulum is a membranous structure containing ribosomes and detoxic enzymes. Golgi complex accumulates and distributes proteins necessary for construction of various structural elements of a cell. Homeostasis is the concept of a dynamic steady state, turnover of bodily substances that maintain physiologic parameters within narrow limits. Stressors cause reactions that alter this dynamic steady state or homeostasis. Deviations from normal values, or homeostasis, cause disease. Necrosis is local cell death and involves the process of cellular self-digestion known as autodigestion or autolysis. As necrosis progresses, most organelles are disrupted and karyolysis, nuclear dissolution from the action of hydrolytic
  • 34. 34 enzymes, becomes evident. There are four major types of necrosis: coagulative, liquefactive, caseous, and fat. Gangrenous necrosis is not a distinctive type of cell death but refers to large areas of tissue death. The two pathways of apoptosis differ in their induction and regulation, and both culminate in the activation of "executioner" caspases. The induction of apoptosis is dependent on a balance between pro- and anti-apoptotic signals and intracellular proteins. The figure shows the pathways that induce apoptotic cell death, and the anti-apoptotic proteins that inhibit mitochondrial leakiness and cytochrome c-dependent caspase activation and thus function as regulators of mitochondrial apoptosis. 6. Control questions of the theme: 1. Cellular injury. Characteristics of the concept of “injury”. Principles of classification of cell injuries. 2. Pathology of signalization. Pathology of signal reception. Disorders of secondary messеngers. 3. Damage to an executive apparatus of a cell. Pathochemical consequences of damage to a cellular nucleus. Proteins of thermal shock, antioncogenes, immediate genes of pre-early reaction. 4. Mechanisms and manifestations of damage to subcellular structures: plasmatic membrane, mitochondria, endoplasmatic reticulum, lysosomes, microtubules and microfilaments, nucleus and cytoplasm. 5. Cellular adaptation to stress. Hypertrophy, hyperplasia, atrophy, metaplasia. Cellular and subcellular regeneration. 6. Molecular mechanisms of cell injury. The role of lipid mechanisms in the pathogenesis of alteration: lipid peroxidation, activation of membrane phospholipids and free fatty acids. Ischemia-reperfusion injury ischemia- reperfusion injury. 7. Causes, mechanisms, and consequences of intracellular calcium concentration increase. 8. Role of electrolyte-osmotic mechanisms in cell injury. Causes, mechanisms, and consequences of disorders of electrolyte transport system in cells. 9. Causes and development of intracellular acidosis in cell injury. 10.Mechanisms of protection and adaptation of cell. Protective and compensative reactions for the renovation of intracellular homeostasis. 11. Antioxidant mechanisms of cells. 12. Mechanisms of apoptosis. The mitochondrial (intrinsic) pathway of apoptosis, the death receptor (extrinsic) pathway of apoptosis, activation, and function of caspases. Autophagy. 13. Necrosis: coagulative, liquefactive, gangrenous, gangrenous, caseous, fat, fibrinoid. 14. Principles of prevention and pathogenetic therapy of cell injury.
  • 35. 35 7. Students’ practical activities Protocol № 6 Date_____________________ Experiment work. To learn the damage of tissues basophile in rat under effect with adrenalin. In the abdominal region of rat enter 5 ml of hemocoel. An animal is killed by ether; good massage a stomach, dissect the abdominal region and collect a liquid in the test-tube. To the got liquid add 0.1% alcoholic solution of neutral red (from the calculation of 0.2 ml paint on 1 ml liquids). Carefully mix and make strokes in which study the initial state of tissue basophile (lens 90, eyepiece10). A liquid which remained is poured out for 1 ml in 2 test tubes: in the first (control) add 0.2 ml of physiological solution, in the second – 0.2 ml (20 μkg) adrenalin. Conclusion:____________________________________________________________ __________________________________________________________________________ __________________________________________________________________________ __________________________________________________________________________ 8. Practice Examination. Practice examination type 1: Choose the correct answer: Test 1. Activation of universal membrane structure injuring mechanism occurs during reperfusion syndrome. This mechanism is referred to as: A. Beta-oxidation of lipids B. Oxidation of cytochromes C. Microsomal oxidation D. Knoop-Lienen cycle E. Peroxidation of lipids Test 2. Which ions accumulation in the cytoplasm of muscular cells accounts for stable constriction of myofibrils? A. Calcium B. Sodium C. Potassium D. Magnesium E. Hydrogen Test 3. Which of the following cellular injury is reversible? A. Hyaline deposits in Liver B. Dystrophic calcification in tissue C. Coagulative necrosis D. Cloudy swelling E. All of the above Test 4. In the process of metabolism in the human organism, the active forms of oxygen are formed, including superoxide anion radical. With the aid of which ferments this anion is inactivated? A. Glucose reductase B. Glucose peroxidase C. Super oxide dimutase D. Catalase E. Peroxidase Test 5. Apoptosis is inhibited by:
  • 36. A. bcl-2 B. p53 С. ras D. c-myc E. p21 Practice examination type 2. Give answer to the questions of the real-life task. The nurse in the Emergency Department is caring for a client who has acute heart failure. The physician is writing orders for pharmacological management, including diuretics. Which laboratory value is most important for the nurse to check before administering medications to treat heart failure? A. Platelet count. B. Potassium. C. Calcium. D. White blood cell count. Answerfor the task:_____________________________________________________ ______________________________________________________________________ ______________________________________________________________________ ______________________________________________________________________ Signature___________________ Literature: Basic: 1. Robbins and Cotran Pathologic Basis of Disease 9thed./ Kumar, Abbas, Fauto. – 2013. –Ch.1.–P.1–28. 2. General and clinical pathophysiology / Ed. by A.V. Kubyshkin – V: NK Publ. – 2011. – P. 134–165. 3. Copstead Lee-Ellen C. Pathophysiology / Lee-Ellen C. Copstead, Jacquelyn L. Banasik // Elsevier Inc, 4th ed. – 2010. – P. 30–84. 4. Pathophysiology, Concepts of Altered Health States, Carol Mattson Porth, Glenn Matfin.– NY, Milwaukee. – 2009. – P. 99–109. 5. Corwin Elizabeth J. Handbook of Pathophysiology / Corwin Elizabeth J. – 3th edition. Copyright В. – Lippincott Williams & Wilkins – 2008. – Chapter 1. – P. 3–35. Additional: 1. Robbins and Cotran Pathologic Basis of Disease 8th ed./Kumar, Abbas, Fauto.–2007.–Chap.1–P. 1–30 2. Gozhenko A.I. General and clinical pathophysiology / A.I. Gozhenko, I.P. Gurcalova // Study guide for medical students and practitioners. Edited by prof. Zaporozan, OSMU. – Odessa. – 2005.– P. 30–41. 3. Essentials of Pathophysiology: Concepts of Altered Health States (Lippincott Williams & Wilkins), Trade paperback (2003) / Carol Mattson Porth, Kathryn J. Gaspard. – Сhapters 1-2. – P. 1–14, 28–35. 4. Silbernagl S. Color Atlas of Pathophysiology / S. Silbernagl, F. Lang // Thieme. NY. – 2000. – P. 2–13. Topic 7: Pathophysiology of the micro- and macrocirculation. 1. The actuality of the theme. Blood vessels function in the delivery of oxygen and nutrients to the tissues and in the removal of waste products from the tissues. Unlike disorders of the respiratory system or central circulation that cause hypoxia and impair oxygenation of tissues throughout the body, the effects of blood vessel disease usually are limited to local tissues supplied by a particular vessel or group of vessels. Disturbances in blood flow can result from pathologic changes in the vessel wall (i.e., atherosclerosis, vasculitides), acute vessel obstruction caused by thrombus or embolus, vasospasm (i.e., Raynaud’s phenomenon), abnormal vessel dilation (i.e., arterial aneurysms or varicose veins), or compression of
  • 37. 37 blood vessels by extravascular forces (i.e., tumors, edema, or firm surfaces such as those associated with pressure ulcers). 2. Duration of the class – 1 hour 30 min. 3. Aim: To know the basic factors of clot formation as the Virchow’s triad: 1. Injury to a vessel’s wall (by mechanic factors, electric current, chemical, biological factors). These abnormalities also accompany atherosclerosis, hypertension, and allergic process. 2. Disturbance of the balance between coagulation and fibrinolytic systems. 3. Slowing of blood flow and its abnormalities. It explains why thrombosis of veins occurs 5 times more often than one of the arteries. To be able: to study typical disorders of microcirculation, their significance, and possible consequences. To perform practical work: to analyze the pathogenesis of sludge formation. 4. Basic level. The name of the previous and future disciplines The receiving of the skills 1. histology 2. biochemistry 3. physiology 4. surgery 5. internal medicine Structure of vascular stream. Vessels, which form the microcirculation stream. Structure and functions of endothelial cells, basal membrane. Two types of nitric oxide (NO) synthesis in endothelium and macrophages. The coagulation cascade. The- central roles of thrombin in hemostasis and cellular activation. The central roles of thrombin in hemostasis and cellular activation. 5. The advice for students. Dynamics of Transcapillar Fluid Exchange (by Starling and Gayton) Active powers Arterial end of capillary mm Hg Equilibrium point mm Hg Venous end of capillary mm Hg Pressing out Capillary hydrostatic pressure 30.0 17.0 10.0 Attractive tissue pressure 5.3 5.3 5.3 Interstitial oncotic pressure 6.0 6.0 6.0 Total 41.3 28.3 21.3 Retentive Oncotic pressure of plasma 28.0 28.0 28.0 Resulting In tissue 13.3 In tissue 0.3 In vessel 6.7 6. Control questions of the theme: 1.Definition of the concept of arterial hyperemia, etiology, and pathogenesis, classification. External symptoms of arterial hyperemia. 2.Changes in tissues caused by arterial hyperemia, their significance, and possible consequences.
  • 38. 38 3.Etiology and pathogenesis of venous hyperemia (congestion), etiology and pathogenesis, external symptoms of venous hyperemia. 4.Changes in tissues caused by venous hyperemia, their significance, and possible consequences. 5.Definition of the concept of ischemia. Etiology, pathogenesis of ischemia. Changes in tissues at ischemia, their significance. Consequences and outcomes of ischemia. External symptoms of ischemia. 6.The concept of reperfusion syndrome, ischemic toxicosis. 7.Definition of stasis. Etiology. Pathogenesis. Capillary (true) stasis. 8.Thrombosis as a reason for peripheral circulation disorder. Etiology and pathogenesis. 9.Embolism as a reason for local circulation disorder. Stages and mechanisms of embolism, types of emboli. The role of reflex mechanisms in the development of common disorders caused by an embolism. 10. Embolism of systemic and pulmonary circulation; embolism of the portal vein. 11. Typical disorders of microcirculation. Intravascular disorders of microcirculation: changes of fluidity and rheological properties of blood. 12. Hemoconcentration, disturbances of suspension stability, aggregation and agglutination of erythrocytes, sludge-phenomenon. 13. The role of aggregation of thrombocytes and disseminated intravascular coagulation in development of microcirculation disorders (DIC-syndrome). 14. Disorders of tone, mechanical safety, and permeability of microvessels. 15. Extracellular disorders of microcirculation. Accumulation of physiologically active substances, ions, edematous fluid in perivascular space. 16. Mesenchymal dystrophy. Capillarotrophic insufficiency. 17. Typical disorders of lymphodynamics: mechanical, dynamical and resorption insufficiency of lymph circulation. 7. Students’ practical activities Protocol № 7 Date_____________________ Experimental work 1. Venous hyperemia and venous stasis of the hand (experiment). Measure up the arterial pressure. Fill the cuff with air so that pressure in it becomes equal to diastolic pressure. Thus arterial blood float will supply active, and venous outflow will decrease. Venous hyperemia will arise. 2 min after it is necessary to pay attention to visible symptoms and complaints due caused by venous hyperemia. Then increase pressure in the cuff up to average mean between systolic and diastolic. Venous stasis will arise. Arterial blood inflow preserves but venous outflow stops. Ask the examined person about his complaints, study skin color, skin temperature and sensitiveness, the condition of
  • 39. 39 superficial veins. Let the air out from the cuff. Note the results of examining in the table and do protocol by a standard scheme. Indexes Venous hyperemia Venous stasis Complaints of the examined Skin colour Skin temperature Skin sensitiveness Condition of superficial veins Conclusion: __________________________________________________________ Experimental work 2. Ischemic stasis and reactive arterial hyperemia of the hand (experiment). To put a cuff, connected to sphygmomanometer on the medial third of shoulder. Measure up the arterial pressure. To fill the cuff with air so that pressure is 10 points higher than systolic pressure. Thus arterial and venous blood flow stops. Ischemic stasis develops. Ask the examined person about his complaints, study skin color, skin temperature and sensitiveness, the condition of superficial veins. Let the air out from the cuff and observe arising from reactive hyperemia. Note the results of examining in the table and do protocol by a standard scheme. Indexes Ischemic stasis Reactive hyperemia Complaints of the inspected Skin color Skin temperature Skin sensitiveness Note the results of examining in the table and does protocol by standard scheme explaining mechanism of every sign arise. Conclusion: __________________________________________________________ Experimental work 3. Fatty embolism of the frog's tongues vessels (experiment). Anesthetize the frog. Fix it on the board with the back down. Cut its chest and open the heart. Put a cotton ball moistened with the solution of Ringer on the heart. Take out the frog's tongue and straighten it upon the opening in the board, watch the circulation of the vessels under the microscope. Carefully inject 0,5-1,0 ml of fat emulsion into the ventricle of the heart. Continue to
  • 40. 40 observe the circulation in the vessels of the tongue. Draw the picture you observed under the microscope. An answer to questions in a discussion: 1. What kind of typical disorders of regional blood circulation is it? 2. What kind of embolism can be concerned the process which appeared during the experiment? 3. Can fatty embolism of the brain appear when big tubular bones are broken? Conclusion: __________________________________________________________ 8. Practice Examination Test 1. Upper neck node of a sympathetic trunk was removed from the rabbit on an experiment. Reddening and increased temperature of the skin of the head are observed. What form of peripheral circulation of the blood developed in the rabbit? A. Venous hyperemia B. Stasis C. Neuroparalytic arterial hyperemia D. Metabolic arterial hyperemia E. Neurotonic arterial hyperemia Test 2. A 54-year-old man was admitted to the hospital with complaints of pain in the right subcostal region, vomiting with blood. Objectively: enlarged liver, varicose veins in the stomach and esophagus. Dysfunction of what vessel is likely to be? A. Vena hepatica B. Vena cava superior C. Vena cava inferior D. Vena porta E. Aorta abdominalis Test 3. What are the mechanisms of aggregate formation? A. Vascular wall damage. B. Blood flow impairment. C. Changes in protein composition of blood. D. Quantitative and qualitative corpuscles. E. All the above-mentioned factors. Test 4. What does the term “sludge” mean? A. Penetration of foreign bodies into the blood and their accumulation in it. B. Formation of a large quantity of immunocomplexes in blood. C. Accumulation and aggregation of blood cells in the microcirculatory bad. D. An extreme degree of corpuscle aggregation. E. Septicemia. Test 5. After fast surgical removing of coronary artery occlusion in a patient with ischemic heart disease, secondary injury of myocardium develop (reperfusion syndrome) characterized by necrobiotic changes in the focus of previous ischemia. This complication results from:
  • 41. 41 A. Excessive accumulation of calcium ions B. Deficiency of potassium ions C. Deficiency of adenosine triphosphate D. Accumulation of hydrogen ions E. Deficiency of creatine phosphate Test 6. A patient has acute pain in his chest, dyspnea, tachycardia, cyanosis, and decreased BP. Pulmonary infarction was diagnosed in this patient. Which factor is the most common cause of pulmonary infarction? A. Congestion in the pulmonary circulation B. Embolism by thrombus from veins of lower extremities C. Increase in number of platelets D. Activation of fibrinolytic system E. Pneumothorax Practice examination type 2. Give answers for the real-life tasks (yes/no) Task 1. Pick out microcirculatory vessels: Indication y/n Indication y/n A Large arteries. E Venules. B Small arteries. F Arteriovenular anastomoses (shunts). C Arterioles. G Veins. D Capillaries. H Aorta Task 2. Which are the regulating mechanisms of microcirculatory vessels: A Reflex. C Hemitic. B Humoral. D Genetic. Task 3. What is true (T), what is false (F) in the following: Questions T F A Intravascular blood clotting (IVBC) can be generalized (disseminated) and local B The process of a platelet plug formation could be divided into two stages: the cellular stage and the plasma stage of coagulation C The stage of adherence, aggregation, and agglutination of platelets and other blood cells is caused by the activating effect of various aggregation stimulators (thrombin, factor of platelets aggregation, serotonin, prostaglandins, thromboxane A2 and others) D The second stage is characterized with the formation of active thrombin, which catalyzes transformation of fibrinogen into fibrin with the formation of a clot Practice examinationtype 3
  • 42. 42 Task 1. What is true (T), what is false (F) in the folowing functional changes for arterial hyperemia Questions T F A Speeding up of blood flow in microcirculation bed B Intensifying of metabolism and organ functioning C Dilatation of small arteries, arterioles, capillaries D Increasing the number of functioning vessels E Hypoxia Task 2. Match the correct answer what is physiological (a) and what is pathologic (b) arterial hyperemia Indication a/b Indication a/b A Infectious rash F Heat hyperemia B Ultraviolet erythema G Systemic lupus erythematosus C Hyperemia of the brain in mental activity H Hyperemia of the brain in meningitis D Hyperemia of pancreatic gland and stomach in digestion J Hyperemia of skeletal muscles in hard physical work E Typhoid fever K Inflammatory hyperemia Task 1. Plaster bandage was imposed on the patient with the right humeral bone fracture. Next day the soft-tissue swelling appeared, extremity became cyanotic, the temperature of the injured hand skin decreased. 1. What violation of circulation of blood arose up? 2. Explain the mechanisms of development of the soft-tissue swelling. 3. What is the mechanism of skin cyanosis? Answersfor the task 1. _________________________________________________ ______________________________________________________________________ ______________________________________________________________________ ______________________________________________________________________ ______________________________________________________________________ Task 2. Alpinists slowly rose on the slope of a mountain during 6 hours. With every step, getting up was given all heavier. A general weakness, palpitation, shortness of breath, syncope, head pain, a decrease in appetite, meteorism ware marked. 1. What was the direct reason for these disorders at alpinists? 2. How is this symptom-complex named? 3. Explain the mechanism of violations development. 4. At what height were alpinists on approximately? 5. What value has tachycardia and tachypnoea in this situation, what are the mechanisms of their development?
  • 43. 43 Answersfor the task 2. _________________________________________________ ______________________________________________________________________ ______________________________________________________________________ ______________________________________________________________________ ______________________________________________________________________ Task 3. Patient V. was in a surgical clinic with thrombophlebitis of right lower extremity. After a careless movement the expressed shortness of breath, pain in a thorax, cyanosis arose up. 1) Are these violations resulted by thrombophlebitis of lower extremity? In what cases are such consequences of thrombophlebitis possible? What did arise up at the patient? Is localization of complications at this patient casual? Explain. 2) Is development of complications of thrombophlebitis possible from the side of other organs - brain, kidney, spleen? Answersfor the task 3. _________________________________________________ ______________________________________________________________________ ______________________________________________________________________ ______________________________________________________________________ ______________________________________________________________________ Signature___________________ Literature: Basic: 1. Robbins and Cotran Pathologic Basis of Disease 9th ed/ Kumar, Abbas, Fauto. –2013.–Ch 3.–P.75–97. 2. General and clinical pathophysiology. Edited by prof. A.V. Kubyskin. Simf. – 2011. – P. 105–120. 3. Pathophysiology, N.K. Symeonova. Kyiv, AUS medicine Publishing. – 2010. – P. 105–125. 4. Copstead Lee-Ellen C. Pathophysiology / Lee-Ellen C.Copstead, J.L.Banasic // Elsev.–2010.–P.347–371. 5. Pathophysiology, Concepts of Altered Health States, C.Mattson Porth,G.Matfin.–NY.–2009.–P.462–474. Additional: 1. Robbins and Cotran Pathologic Basis of Disease 8th ed./Kumar, Abbas, Fauto. –2007.–Ch.4.–P. 81–100. 2. Gozhenko A.I. General and clinical pathophysiology / A.I. Gozhenko, I.P. Gurcalova // Study guide for medical students and practitioners. Edited by prof. Zaporozan, OSMU. – Odessa. – 2005.– P. 60–67. 3. Essentials of Pathophysiology: Concepts of Altered Health States (Lippincott Williams & Wilkins), Trade paperback (2003) / Carol Mattson Porth, Kathryn J. Gaspard. Chapters 15. – P. 263 – 272, 178–189. Topic 8: Inflammation: phlogogenic factors, pathogenesis of alteration, local and general sings. Kongeim’s experiment. 1. The actuality of the theme. Inflammation is the reaction of vascularized tissue to local injury. The causes of inflammation are many and varied. Inflammation commonly results because of an immune response to infectious microorganisms. Other causes of inflammation are trauma, surgery, caustic chemicals, extremes of heat and cold and ischemic damage to body tissues. Inflammatory conditions are named by adding the suffix – itis to the affected organ or system. For example, appendicitis refers to inflammation of the appendix, pericarditis to inflammation
  • 44. 44 of the pericardium, and neuritis to inflammation of a nerve. More descriptive expressions of the inflammatory process might indicate whether the process was acute or chronic and what type of exudate was formed (e.g., acute fibrinous pericarditis). 2. Duration of the class – 1h 30 min. 3. Aim: To know: 1. Definition of the notion "inflammation". 2. Causes of the inflammation. 3. The role of mediators in inflammation development. 4. The meaning of the organism reactivity in inflammation development. 5. General manifestation of inflammatory reaction. 6. Clinical symptoms of the inflammation. 7. Causes and mechanisms of the vascular permeability disorder in inflammation. 8. Methods of the vascular permeability study in the area of inflammation. To be able: - to describe the processes of alteration, exudation and proliferation; - to give description of mediators of inflammation and explain their role in pathogenesis of inflammation; - to reproduce inflammation in an experiment. To perform practical work: to analyse of the pathogenesis of inflammation: disorders and after effect. 1. Basic level. The name of the previous and future disciplines The receiving of the skills 1. Histology 2. biochemistry 3. physiology 4. internal medicine 5. surgery Functional parts of bloodstream. Conception about the microcirculation. Mechanisms of regulation of blood circulation in capillaries and venules. Conception about the role of connecting tissue cells, their structure, main functions. Conceptis about the products ofarachidonic acid cascade, kalikrein-kinin system of blood, complement system 5. The advice for students . Inflammation hallmarks Systemic Local Leukocytosis  redness (rubor)  heat (calor)  pain (dolor)  swelling (tumor)  loss of function (functio laesa) Leukopenia (in inflammation of viral origin) Fever Change of protein composition of blood: acute phase α- protein (acute inflammation) and -globulins (chronic inflammation) Change of ferment composition of blood: increase of transaminase, hyaluronidase, trombokinase activity Increase of erythrocyte sedimentation Change of hormone content:  catecholamins  corticosteroids
  • 45. 45 Immune system alterations and allergization of organism:  antibody titre  appearance of sensibilizing lymphocytes in blood  development of allergic reactions 6. Control questions of the theme: 1.Definition of the concept “inflammation”. Etiology of inflammation. Classification of the inflammatory agents. 2.Stages of inflammation. Cardinal symptoms of inflammation in experiment (Cels, Galen). Classification of inflammation. 3.Acute inflammation. Primary and secondary alteration. Causes and mechanisms of secondary alteration. 4.Mediators of inflammation. Their classification. Role of cytokines in pathogenesis of inflammation. 5.Role of lysosomal enzymes, free radicals, peroxides and system of complement in tissue injury. 6.Complement activation in the development of inflammatory process. Its disorders. 7.Products of tissue basophile degranulation. 8.Derivates of arachidonic acid: prostaglandins, leukotrienes, thromboxanes. 9.Kallikrein-kinin system. 10.Biochemical and physicochemical disorders in focus of inflammation. Local acidosis, hyperosmia, hyperoncia. 11.Methods of study of vascular permeability are in the hearth of inflammation. 12.Cellular events: leukocyte recruitment and activation. Endothelial and leukocyte adhesion molecules. 7. Students’ practical activities Protocol № 8 Date_____________________ Experimental work. The change of the vessels permeability for anaphylaxis-like inflammation (experiment). It is necessary to inject trypan blue under the back skin of the rat in the quantity of 1-2 ml. 20 minutes after causing the inflammation on the rear extremity using the method mentioned above. After some time examine the skin color change of the animal and also compare the color of injured and opposite extremities. Analyse the mechanism of the permeability change. Write down the experiment results and protocol of experiment according to a scheme. For a discussion of the results, it is necessary to give an answer to questions. 1. Enumerate local signs of inflammation. 2. What do biological active substances appear in the area of inflammation?
  • 46. 46 3. How and why the permeability of the vessels in the area of the inflammation does change? 4. What changes in the metabolism to promote the development of the inflammation signs? 5. What meaning does inflammation have for an organism? Conclusion: ____________________________________________________________ ______________________________________________________________________ ______________________________________________________________________ ______________________________________________________________________ 8. Practice Examination. Practice examination type 1: Choose the correct answer: Test 1. Blood plasma of a healthy man contains several dozens of proteins. During an illness new proteins can originate, namely the protein of "acute phase". Select such protein from the listed below: A. G immunoglobulin B. Prothrombin C. C-reactive protein D. Fibrinogen E. A immunoglobulin Test 2. Which one of the following enzymes is the major component of granulocytes producing antibacterial activity during acute inflammation? A. Myeloperoxidase B. Acid hydrolase С. Protease D. Lysozyme Test 3. Humoral mediators of inflammation include: А. Histamine В. Interleukine С. Prostaglandin Е2 D. Serotonin Е. Bradykinin Test 4. The first cells which migrate in area of inflammation are: A. Neutrophiles B. Lymphocytes C. Monocytes D. Eosinophiles E. Basophiles Test 5. Necrosis focus was observed in the area of hyperemia and skin edema in a few hours after burn. What mechanism strengthens destructive effects in the inflammation area? A. Proliferation of fibroblasts B. Diapedesis of erythrocytes C. Primary alteration D. Secondary alteration E. Emigration of lymphocytes Test 6. Prostaglandins (PGs) have effects on a variety of tissues. The different prostaglandins may have different effects. Which of the following is not correct statement? A. The human arteriolar smooth muscle is relaxed by PgE2 and PgI2 where as TxA2 and PgF2α cause vasoconstriction. B. PgE2 has marked oxytocic action while PgF2α has tocolytic action.